Download as pdf or txt
Download as pdf or txt
You are on page 1of 72

Contents

Mathematical competitions in Croatia in 2015 1

I.1 First round – City competition . . . . . . . . . . . . . . . . . . . . . . . . 2

I.2 Second round – County competition . . . . . . . . . . . . . . . . . . . . . 5

I.3 Final round – National competition . . . . . . . . . . . . . . . . . . . . . 8

I.4 Croatian Mathematical Olympiad . . . . . . . . . . . . . . . . . . . . . . 23

Mathematical competitions in Croatia in 2016 37

II.1 First round – City competition . . . . . . . . . . . . . . . . . . . . . . . . 38

II.2 Second round – County competition . . . . . . . . . . . . . . . . . . . . . 41

II.3 Final round – National competition . . . . . . . . . . . . . . . . . . . . . 44

II.4 Croatian Mathematical Olympiad . . . . . . . . . . . . . . . . . . . . . . 57

In this booklet we present the problems from mathematical competitions for


category A in Croatia in 2015 and 2016. The solutions are given for the problems of
the Final round and the Croatian Mathematical Olympiad.
Problems are numerated according to secondary school classes, e.g. 2.4. is the 4th
problem for the 2nd class of secondary school (10th grade).
We have tried to acknowledge the authors or the sources of the problems wherever it
was possible.

c Croatian Mathematical Society, 2016

Editor:
Matija Bašić

Translation:
Nikola Adžaga, Petar Bakić, Mea Bombardelli,
Ilko Brnetić, Marijan Polić, Kristina Ana Škreb,
Azra Tafro, Stipe Vidak, Vanja Wagner

Figures and typesetting:


Ivan Kokan

Printed in:
Element Ltd.
June 2016, Zagreb, Croatia

Published in 240 copies


Mathematical competitions in Croatia in 2015

In Croatia, children undergo twelve years of formal education – from 1st to 8th grade
in primary school and from 9th to 12th grade in secondary school. Mathematical com-
petitions are held for students from 4th to 12th grade. Competitions at the secondary
school level have two categories – category A (for students who attend mathematical
grammar schools) and category B (for all other secondary school students).
In the year 2015, 16881 students participated in the first round of the competitions,
among them 1050 in category A for secondary schools. The City competitions were
held on 29th of January. All tests at this level consist of five easier problems and two
harder problems, as well as tests for category B at County competitions. All other
tests at County competitions and all the tests at National competition consist of five
problems.
On County competitions held on 27th of February, 2198 primary school students
and 1246 secondary school students (539 of them in category A) participated. The
best 83 students from primary school (from 5th to 8th grade), and 163 secondary
school students (91 in category A and 72 in category B) were invited to the National
competition in Trogir (from 8th to 10th of April).
After the National competition, an additional three-round Team selection process,
called Croatian Mathematical Olympiad, was organised. The top 19 students from
category A in the National competition took two tests on 18th and 19th of April.
Separate final selection tests for the IMO and MEMO teams took place on 25th
of April. All tests consisted of four olympiad-style problems from four different areas:
algebra, combinatorics, geometry and number theory.
After the Croatian Mathematical Olympiad, the teams for the 56th International
Mathematical Olympiad (IMO) in Chiang Mai, Thailand and the 9th Middle
European Mathematical Olympiad (MEMO) in Koper, Slovenia were selected:

IMO MEMO
1
Adrian Beker (grade 10) Domagoj Bradač 1 (grade 11)
Ivan Lazarić 2 (grade 12) Josip Kelava 5 (grade 10)
Lukas Novak 3 (grade 10) Andrija Mandić 1 (grade 11)
Petar Orlić 1 (grade 11) Petar Nizić-Nikolac 1 (grade 9)
Daniel Paleka 4 (grade 11) Patrik Papac 6 (grade 10)
Kristijan Štefanec 1 (grade 12) Leon Starešinić 1 (grade 11)

1
XV. gimnazija, Zagreb, 2 Gimnazija Pula, Pula, 3 Gimnazija Josipa Slavenskog,
Čakovec, 4 Gimnazija Franje Petrića, Zadar, 5 Prva gimnazija Varaždin, Varaždin,
6
Gimnazija Dubrovnik, Dubrovnik.

1
I.1 First round – City competition

1.1. Determine positive integer n such that the sum of his two smallest divisors is 6
and the sum of his two largest divisors is 1122. (Function Space)

1.2. Let a and b be positive real numbers such that a3 + b3 = 2ab(a + b).
a2 b2
Determine + . (Ratko Višak)
b2 a2

1.3. The area of the intersection of a smaller and larger square equals two thirds of
the area of the smaller square, as well as one fifth of the area of their union.
Determine the ratio of the sides of the smaller and larger square.
(Andrea Aglić-Aljinović)

1.4. There are 100 circles in the plane, all having the same centre with radii 1, 2, . . . ,
100. The smallest circle is coloured in red and each of the 99 annuli bounded by
two circles is coloured in red or green, such that adjacent regions are of different
colour. Determine the total area of green regions. (AIME 2003)

1.5. Let I be the incentre of the acute triangle ABC and let |AC| > |BC|. The angle
bisector and the altitude from vertex C close an angle of 10◦ . If ^AIB = 120◦ ,
determine the angles of the triangle ABC. (Ilko Brnetić)

1.6. Does there exist a positive integer n such that n2 + 2n + 2015 is a perfect
square? (Ilko Brnetić)

1.7. Five teams are playing at a football tournament, each playing with the other
exactly once. Each victory is worth 3 points, defeat 0 points and a draw 1 point.
Is it possible that at the end of the tournament, in the final ranking, each team
except the last has exactly two points more than the next one? (Ilko Brnetić)

∗∗∗
Ä ä
2.1. Prove the following statement: if z is a complex number such that Re z−i
z+i = 0,
then |z| = 1. (Ilko Brnetić)

2.2. Let O be the circumcentre of the acute triangle ABC and let N be the foot of
the altitude from point A. Prove that ^BAN = ^CAO. (Hong Kong)

2.3. Determine the number of positive integers smaller than 1 000 000, that are also
perfect squares and give a remainder 4 when divided by 8. (Ilko Brnetić)

2
First round – City competition 3

2.4. The sum of squares of all solutions of the equation x4 + ax2 + b = 0 is 32, and
the product of all solutions of that equation is 4. Determine a and b.
(Tamara Srnec)

2.5. Let ABCD be a quadrilateral such that |AB| = 6, |BC| = 9, |CD| = 18 and
|AD| = 5 hold. Determine the length of the diagonal AC if it is known that it
is a positive integer. (Andrea Aglić-Aljinović)

2.6. A field of the shape of a circular sector needs to be fenced using a wire of
length d. What is the maximum area of that field? (Ilko Brnetić)

2.7. Every team at a basketball tournament is playing with every other team exactly
twice. Each victory is worth 2 points, defeat 0 points and there are no draws.
Determine all positive integers n such that there exists a basketball tournament
with n teams with the overall winner having 26 points and exactly two teams
having the lowest number of points 20. (Ilko Brnetić)

∗∗∗

3.1. For any two positive real numbers x and y prove

log2 (xy) > log(x2 ) log(y 2 ). (Ilko Brnetić)

3.2. Determine all values that the expression

1 + cos2 x 1 + sin2 x
+ ,
sin2 x cos2 x
can attain, where x is a real number. (Tamara Srnec)

3.3. Determine the largest positive integer n such that

n + 5 | n4 + 1395. (Ilko Brnetić)


3.4. Let ABCD √ be a tetrahedron such that ^BAC = ^CAD = ^DAB = 90 ,
|AD| = 2 2 and |AB| = |AC| = 3 holds. Determine the radius of the inscribed
sphere of the tetrahedron. (Mea Bombardelli)

3.5. Determine the least positive integer n such that in every set consisting of n
integers there are three pairwise distinct elements a, b and c such that ab+bc+ca
is divisible by 3. (Ilko Brnetić)

3.6. Let ABC be a triangle in which tan ^BAC = 1 and tan ^ABC = 2 holds.
Determine the ratio |BC| : |AB|. (Ilko Brnetić)
4 Mathematical competitions in Croatia in 2015

3.7. There are ten white, and one red, blue, green, yellow and purple card. White
cards are identical. On exactly one side of each card is the sign X. In how many
ways is it possible to put the cards one on another such that no two cards face
each other with the side having the sign X? (Matija Bašić)

∗∗∗

4.1. Let a, b and c be real numbers and let f : R → R be a function given by

f (x) = ax5 + bx3 + c sin x − 1.

If f (−2015) = 2015, determine f (2015). (Stipe Vidak)

10
4.2. Determine the coefficient of x9 in the polynomial 1 + x3 + x6 .
(Ratko Višak)

n
X Sn + 1
4.3. Let n be a positive integer and let Sn = k!(k 2 + k + 1). Determine .
(n + 1)!
k=1
(Ratko Višak)

4.4. In a box, there is one red and five white balls marked by numbers 1, 2, 3, 4
and 5. Without looking, Domagoj draws one ball at a time until he draws
the red ball and then he stops. Drawn balls are not returned into the box.
What is the probability that the sum of the numbers on the drawn balls is at
least 10? (Ilko Brnetić)

4.5. If n is a positive integer, let Rn denote the integer that in the decimal repres-
entation consists of n digits 1. Prove the following statement: if Rn is a prime
number, then n is a prime number, too. (Mathematics Stack Exchange)

4.6. Let M and N be, respectively, the feet of the altitudes from vertices A and B of
the acute triangle ABC. Let Q be the midpoint of the segment M N and let P
be the midpoint of the segment AB. If |M N | = 10 and |AB| = 26, determine
the length of the segment P Q. (Hong Kong)

4.7. Let n be a positive integer. Each of the numbers n, n + 1, n + 2, . . . , 2n − 1


has a largest odd divisor. Determine the sum of these largest odd divisors.
(Mathematics Stack Exchange)
I.2 Second round – County competition

1.1. Let x and y be distinct real numbers such that 2xy + 1 6= 0, and let
6x2 y 2 + xy − 1 x(x2 − 1) − y(y 2 − 1)
A= and B = .
2xy + 1 x−y

Determine which number is larger, A or B. (Tonći Kokan)

1.2. Positive integers a, b and prime number p satisfy the equation a2 + p2 = b2 .


Prove that 2(b + p) is a perfect square. (Mathematics Stack Exchange)

1.3. Determine how many 6-digit positive integers are there such that by removing
its first two digits and its last two digits we get two 4-digit numbers, which give
the same remainder when divided by 99. (Mea Bombardelli)

1.4. Let AC be the diameter of the circle k1 with the centre B. Circle k2 touches
the line AC at the point B and the circle k1 at the point D. Tangent from A
(different from AC) to circle k2 touches that circle at the point E and intersects
the line BD in the point F . Determine the ratio |AF | : |AB|. (Hong Kong)

1.5. For a positive integer n, we say that an array with three rows and n columns is
magical if there exists a positive integer k, 1 6 k 6 n, such that
• the first row is filled with numbers . . . 1, 2, . . . , n in that order
• the second row is filled with numbers . . . k, k + 1, . . . , n, 1, 2, . . . , k − 1 in
that order
• the third row is filled with numbers between 1 and n in such order that
the sums of the three numbers in each column are all the same.
Determine all positive integers n for which a magical array exists, and for each
such n determine the number of magical arrays. (Ireland)

∗∗∗

2.1. Determine all pairs (a, b) of integers such that the intersections of the parabola
y = x2 + ax + b and the coordinate axes form a triangle whose area is equal
to 3. (Nikola Adžaga)

2.2. Determine all triples (a, b, c) of real numbers such that


1
a2 + b2 + c2 = 1 and (2b − 2a − c)a > . (Ivan Kokan)
2

5
6 Mathematical competitions in Croatia in 2015

2.3. Determine all quadruples (a, b, c, d) of positive integers such that

a3 = b2 , c 5 = d4 and a − c = 9. (China)

2.4. Let O be the circumcentre and H be the orthocentre of the triangle ABC.
Ray AO intersects the circumcircle of the triangle ABC in point D. Prove that
the line HD passes through the midpoint of the segment BC. (Folklore)

2.5. There are 4 easy and 8 difficult problems on a mathematical competition. There
are n students participating on this competition and each student has correctly
solved exactly 11 out of 12 problems.
For each pair of a difficult and easy problem, the number of students that have
correctly solved both problems was written. The sum of all 32 written numbers
is 256. Determine n. (Canada 2011)

∗∗∗

3.1. Let I be the incentre of the triangle ABC. If |AI| = |BC| and ^ACB =
2^BAC, determine the angles of the triangle ABC. (Brazil 2011)

3.2. Let bxc denote the largest integer not greater than real number x.
Determine all real solutions of the equation

11 bxc + x + 12 = 9x.
 
(Mario Krnić)

3.3. Let n be a positive integer larger than 1 such that both 2n − 1 and 3n − 2 are
perfect squares. Prove that 10n − 7 is composite. (Problem-Solving Strategies)

3.4. Let ABCD be a convex quadrilateral such that ^BAD = 50◦ , ^ADB = 80◦
and ^ACB = 40◦ holds. If ^DBC = 30◦ + ^BDC, determine ^BDC.

3.5. Marko has 2n cards (n ∈ N), two cards with each of the numbers 1, 2, . . . , n.
Once we shuffled and put them one on top the other, he noticed that for every k
in {1, 2, . . . , n} between two cards with the number k there are exactly k other
cards. Prove that n2 + n is divisible by 4. (Counting and Configurations)

∗∗∗

4.1. Let a = 2015 2015 and (an ) be the sequence such that a1 = a and an+1 = aan
for n > 1. Does there exist a positive integer n such that an > 2015?

4.2. One side of a square lies on the line y = 2x−17, while the remaining two vertices
lie on the parabola y = x2 . Determine the area of that square. (China 2005)
Second round – County competition 7

4.3. Let n be a positive integer and a0 , a1 , . . . , a2n ∈ − π2 , π2 real numbers such that

tan ak = 2k−n for k = 0, 1, . . . , 2n.

Determine the sum a0 + a1 + · · · + a2n . (Matija Bašić)

4.4. A positive integer is called wacky if its decimal representation contains 100
digits, and if by removing any of those digits one gets a 99-digit number divisible
by 7. How many wacky positive integers are there? (Stipe Vidak)

4.5. A finite number of real numbers are ordered around a circle, and each number
is coloured red, white or blue. Each red number is twice smaller than the sum
of its two adjacent numbers, each white number is equal to the sum of its two
adjacent numbers, and each blue number is twice larger than the sum of its two
adjacent numbers. Let w be the sum of all white numbers, and b be the sum of
all blue numbers, where both w and b are not 0.
w
Determine the ratio . (Ukraine 2013)
b
I.3 Final round – National competition

1.1. Ten chairs are arranged around a round table and marked with numbers 1 to 10
successively (in such a way that chairs 1 and 10 are also adjacent), and a knight
is sitting in each chair. In the beginning, every knight has an even number of
coins. Simultaneously, each knight gives half of his coins to his left neighbour,
and the other half to his right neighbour. After that, the knight sitting in chair 1
has 22 coins, and each succeeding knight has two more coins, up until the knight
in chair 10 that has 40 coins.
How many coins did the knight that ended up with 36 coins have in the
beginning? (Hong Kong)

1.2. Prove that there does not exist a positive integer n such that 7n − 1 is divisible
by 6n − 1. (Ilko Brnetić)

1.3. Let a, b and c be positive real numbers such that a2 + b2 + c2 = 3. Prove that
a4 + 3ab3 b4 + 3bc3 c4 + 3ca3
+ + 6 4. (Tonći Kokan)
a3 + 2b3 b3 + 2c3 c3 + 2a3

1.4. The first n positive integers are written on a board (n > 3). Ante repeats the
following procedure: first he chooses two numbers on the board, and then he
increases them both by the same arbitrary positive integer.
Determine all positive integers n such that Ante can, by repeating this proced-
ure, achieve that all numbers on the board are equal. (Ilko Brnetić)

1.5. Circles k1 and k2 intersect in points A and B. Line l intersects circle k1 in


points C and E, and circle k2 in points D and F in such a way that D is
between C and E, and E is between D and F . Lines CA and BF intersect in
point G, and lines DA and BE in point H.
Prove that CF k HG. (Geometry from the Land of the Incas)

∗∗∗

2.1. Let a, b, c and d be four pairwise distinct real numbers. If a and b are solutions
of the equation x2 − 10cx − 11d = 0, and c and d are solutions of the equation
x2 − 10ax − 11b = 0, determine the sum a + b + c + d.
(Mathematics Stack Exchange)

2.2. Determine all triples (p, m, n) of positive integers such that p is a prime number
and
pm − n3 = 8. (Greece)

8
Final round – National competition 9

2.3. Let ABC be an acute triangle with |AC| > |AB|. Let N be the foot of the
altitude from point A to side BC. Let P be a point on the extension of AB
over point B, and let Q be a point on the extension of AC over point C such
that BP QC is a cyclic quadrilateral.
If |N P | = |N Q|, show that N is the centre of the circle circumscribed to triangle
AP Q. (Japan)

2.4. Let a, b and c be positive real numbers such that a + b + c = 1. Prove that
a b c 1 1 1 1
Å ã
+ + 6 + + . (Tonći Kokan)
a + b2 b + c2 c + a2 4 a b c

2.5. A grasshopper is sitting in the origin of the number line, at number 0, and then
it jumps, always in the same direction. For a positive integer k, in the first
jump the grasshopper jumps to number 1, and every following jump is exactly
k times longer than the previous jump. There is a hole in place of all multiples
of number 2015.
Determine all positive integers k such that the grasshopper can jump 2015 times
without falling into a hole. (Ilko Brnetić)

∗∗∗

3.1. In a triangle ABC we have |BC| + |AC| = 2|AB| and ^BAC − ^CBA = 90◦ .
Determine the cosine of the angle ^ACB. (Hong Kong)

3.2. Determine all triples (p, m, n) of positive integers such that p is prime and
2m p2 + 1 = n5
holds. (Finland)

3.3. In a country between every two cities there is a direct bus or a direct train line
(all lines are two-way and they don’t pass through any other city). Prove that
all cities in that country can be arranged in two disjoint sets so that all cities
in one set can be visited using only train so that no city is visited twice, and
all cities in the other set can be visited using only bus so that no city is visited
twice. (L. Gerencsér, A. Gyárfás, On Ramsey-type problems)

3.4. On the side AC of the triangle ABC the points D and E are given such that D
is between C and E. Let F be the intersection of the circumcircle of the triangle
ABD and the line through the point E parallel to BC such that E and F are
on different sides of the line AB. Let G be the intersection of the circumcircle
of the triangle BCD and the line through E parallel to AB such that E and G
are on different sides of the line BC.
Prove that the points D, E, F and G lie on the same circle.
(Geometry from the Land of the Incas)
10 Mathematical competitions in Croatia in 2015

3.5. Let a, b and c be positive real numbers such that a + b + c > 1. Prove that
a − bc b − ca c − ab 3
+ + 6 . (Canada 2008)
a + bc b + ca c + ab 2

∗∗∗

4.1. Determine all functions f : R → R such that for all real numbers x and y

f (xy)(x + f (y)) = x2 f (y) + y 2 f (x)

holds. (Tonći Kokan)

4.2. Let ABC be a right triangle with the right angle at C. Let A0 , B 0 and C 0 be
the pedals of the perpendiculars from the centroid of the triangle ABC onto the
lines BC, CA and AB respectively.
Determine the ratio of the areas of the triangles A0 B 0 C 0 and ABC.

4.3. Determine all positive integers n for which there exists a divisor d of n such
that
dn + 1 | d2 + n2 .

4.4. Let n be a positive integer. Determine all positive real numbers x for which

22 32 (n + 1)2 n(n + 3)
+ + ··· + + nx2 = nx +
x+1 x+2 x+n 2
holds. (Tonći Kokan)

4.5. On the board 8 × 8 tromino-tiles of the shape have to be placed in such a


way that each tile covers exactly three cells of the board and the tiles cannot
overlap.
What is the least possible number of tromino-tiles that one can place on the
board so that no additional tromino-tile can be placed afterwards?
(Counting and Configurations)
Final round – National competition 11

I.3.1 Solutions

1.1. Let us denote by 2x1 , 2x2 , . . . , 2x10 the number of coins that the knights sitting
in chairs 1, 2, . . . , 10 had in the beginning, respectively. We have to determine 2x8 .
We have a system of equations: x10 +x2 = 22, x1 +x3 = 24, x2 +x4 = 26, x3 +x5 = 28,
. . . , x8 + x10 = 38, x9 + x1 = 40.
By combining those equations we get

x10 = 38 − x8 ,
x2 = 22 − x10 = 22 − (38 − x8 ) = x8 − 16,
x4 = 26 − x2 = 26 − (x8 − 16) = 42 − x8 ,
x6 = 30 − x4 = 30 − (42 − x8 ) = x8 − 12,
x8 = 34 − x6 = 34 − (x8 − 12) = 46 − x8 .

From the last equation it follows that 2x8 = 46, so the knight that ended up with 36
coins had 46 coins in the beginning.

1.2. Assume that such n exists.


Note that 5 divides 6n − 1 = (6 − 1)(6n−1 + · · · + 1), so 5 also has to divide 7n − 1.
The powers of 7 when divided by 5 give remainders 2, 4, 3, 1, . . . and these remainders
repeat periodically. Therefore, 7n − 1 will be divisible by 5 if and only if n is divisible
by 4, i.e. n = 4k.
By the formula for the difference of k th powers, we can conclude that 64 − 1 divides
64k − 1. Moreover, 7 divides 64 − 1 = (62 − 1)(62 + 1) = 35 · 37 = 5 · 7 · 37. Therefore,
7 divides 6n − 1, so it also divides 7n − 1, which is a contradiction.

1.3. Note that


a4 + 3ab3 a(a3 + 2b3 ) + ab3 ab3
= = a + .
a3 + 2b3 a3 + 2b3 a3 + 2b3

By the inequality between arithmetic and geometric means we have



3
a3 + 2b3 = a3 + b3 + b3 > 3 a3 b3 b3 = 3ab2 .

We can conclude that


a4 + 3ab3 ab3 ab3 b
3 3
=a+ 3 3
6a+ =a+ .
a + 2b a + 2b 3ab2 3
Analogously, we have

b4 + 3bc3 c c4 + 3ca3 a
6b+ and 6c+ ,
b3 + 2c3 3 c3 + 2a3 3
12 Mathematical competitions in Croatia in 2015

so by adding the inequalities we have

a4 + 3ab3 b4 + 3bc3 c4 + 3ca3 4


3 3
+ 3 3
+ 3 3
6 (a + b + c).
a + 2b b + 2c c + 2a 3

Finally, by using the inequality between arithmetic and quadratic means we have

a+b+c a2 + b2 + c2
4· 64· = 4.
3 3
This finishes the proof.

1.4. Assume n = 4k. Then Ante can achieve that all numbers on the board are
equal in the following way: he will increase by 1 the numbers 1 and 3, 5 and 7, . . . ,
4k − 3 and 4k − 1. By doing that, he gets that the numbers on the board are all even
numbers smaller than or equal to n, and each is written twice. Finally, he increases
2 and 2 by n − 2, 4 and 4 by n − 4, . . . , n − 2 and n − 2 by 2, and he gets that all
numbers on the board are equal to n.
Assume n = 2k +1. Then Ante can achieve that all numbers on the board are equal in
the following way: he will increase by 1 the numbers 1 and n, 3 and n, . . . , n−2 and n.
By doing that, he gets that the numbers on the board are all even numbers smaller
3n − 1
than or equal to n, each written twice, and the number . Finally, he increases
2
3n − 5 3n − 9 n+1
2 and 2 by , 4 and 4 by , . . . , n−1 and n−1 by . Now all numbers
2 2 2
3n − 1
on the board are equal to .
2
Assume n = 4k + 2. Then Ante cannot achieve that all numbers are equal. The sum
of all numbers on the board is initially odd, because

n(n + 1)
1 + 2 + ··· + n = = (2k + 1)(4k + 3).
2

Since there is an even number of numbers on the board, if they were equal their sum
would be an even number. On the other hand, in each step the sum of numbers on
the board is increased by an even number, so the sum will never be even.
Therefore, Ante can achieve that all numbers are equal if and only if n is not of the
form 4k + 2, k ∈ N.

1.5. It suffices to show that ^ECA = ^HGA.


Since the quadrilateral ACBE is cyclic, we have ^ECA = ^EBA, so it suffices to
show that ABGH is a cyclic quadrilateral.
From triangle DEH we have ^DHE = 180◦ − ^EDH − ^HED, i.e.

^AHB = ^DHE = 180◦ − ^F DA − (180◦ − ^CEB),


Final round – National competition 13

so by using that ^CEB = ^CAB (which holds because ACBE is also cyclic) we get
that

^AHB = 180◦ − ^F DA − (180◦ − ^CAB) = 180◦ − ^F DA − ^BAG.

H
G
A
C

E
D
F
l

k2
k1 B

Quadrilateral ADBF is cyclic, so we have ^F DA = ^F BA = ^GBA. It follows


that

^AHB = 180◦ − ^F DA − ^BAG = 180◦ − ^GBA − ^BAG = ^AGB.

Therefore, ABGH is a cyclic quadrilateral, which finishes the proof.

∗∗∗

2.1. From Vieta’s formulae we have a + b = 10c and c + d = 10a. By adding these
equations we get
a + b + c + d = 10(a + c).

Since a is a solution of the equation x2 − 10cx − 11d = 0, and d = 10a − c, it follows


that

0 = a2 − 10ac − 11d = a2 − 10ac − 11(10a − c) = a2 − 110a + 11c − 10ac.

Analogously, we get
c2 − 110c + 11a − 10ac = 0.

By subtracting these equations it follows that

(a − c)(a + c − 121) = 0.

Since a 6= c, we get a + c = 121. Therefore, a + b + c + d = 10 · 121 = 1210.


14 Mathematical competitions in Croatia in 2015

2.2. By moving n3 , we get a sum of cubes on the right-hand side:


pm = n3 + 8 = (n + 2)(n2 − 2n + 4).

Since p is prime, each of the factors on the right-hand side must be a power of p:
n + 2 = pα , n2 − 2n + 4 = pβ ,
where α and β are obviously positive integers.
Note that n2 −2n+4 > n+2, since that is equivalent to the claim that n2 −3n+2 > 0,
i.e. (n − 1)(n − 2) > 0, which holds because n is a positive integer. Therefore, β > α.
We can conclude that pα divides both n + 2 and n2 − 2n + 4, so it also divides
n · (n + 2) − (n2 − 2n + 4) = 4n − 4,
and then it also divides 4 · (n + 2) − (4n − 4) = 12, so p = 2 or p = 3.
If p = 2, then n + 2 = pα is at most 4, and since n > 0, it follows that n = 2, which
gives the solution (2, 4, 2).
If p = 3, then n + 2 = pα is 3, so n = 1, which gives the second solution (3, 2, 1).

2.3. The sum of opposite angles in a cyclic quadrilateral is 180◦ , therefore ^CQP =
180◦ − ^P BC and ^QP B = 180◦ − ^BCQ, from which it follows that ^AQP =
^CBA and ^QP A = ^ACB.
Let O be the centre of the circle circumscribed to AP Q. Then ^AOP = 2^AQP .
Since the triangle AOP is isosceles, we have that ^P AO = 90◦ − ^AQP = 90◦ −
^ABC.
A A

B C B C
O N

Q Q

P P p

Let p be the line which closes an angle of 90◦ − ^ABC with line AP , and which
intersects the segment P Q. Since ^AN B = 90◦ , it follows that ^AN B = 90◦ −
^ABC, which implies that A, N and O all lie on p.
Since |AB| = 6 |AC|, it follows that ^AQP = ^ABC 6= ^ACB = ^AP Q, i.e. A
does not lie on the bisector of segment P Q. Therefore, the intersection of line p and
bisector of P Q is unique and it follows that N = O.
Final round – National competition 15

2.4. By using the condition a + b + c = 1 and the inequality between arithmetic and
geometric means, we have that
a a a a 1
= = 2 6 = .
a + b2 a(a + b + c) + b2 a + b2 + ab + ac 2ab + ab + ac 3b + c

By applying the inequality between harmonic and arithmetic means, it follows that
4 4 b+b+b+c 3b + c
3 1 = 1 1 1 1 6 = ,
b + c b + b + b + c
4 4

i.e.
a 1 1 3 1
Å ã
6 6 + .
a + b2 3b + c 16 b c
Analogously,
b 1 3 1 c 1 3 1
Å ã Å ã
6 + and 6 + .
b + c2 16 c a c + a2 16 a b

Finally, by adding the inequalities above, we have that


a b c 1 4 4 4 1 1 1 1
Å ã Å ã
+ + 6 + + = + + .
a + b2 b + c2 c + a2 16 a b c 4 a b c

2.5. Let an be the number the grasshopper is located at after the nth jump, i.e.

a1 = 1, an = 1 + k + · · · + k n−1 , n > 2.

We are looking for all numbers k such that 2015 - an for all n = 1, . . . , 2015.
Suppose that M (k, 2015) = d > 1. Then every an divided by d gives the remainder 1,
and since 2015 is divisible by d we have that 2015 - an for all n. Therefore, all positive
integers which are not relatively prime to 2015 comply with the terms of the problem.
If M (k, 2015) = 1, we observe the remainders of dividing a1 , . . . , a2015 by 2015. If one
of them is divisible by 2015, such a k is not good. Otherwise, since there are 2014
possible remainders, at least two numbers give the same remainder. Let these numbers
be al and am , m > l. In this case, their difference is divisible by 2015. On the other
hand, we have that

am − al = k l + · · · + k m−1 = k l (1 + · · · + k m−l−1 ) = k l · am−l .

From 2015 | k l · am−l and M (k, 2015) = 1, it follows that 2015 | am−l , which is in
contradiction with the assumption that none of the numbers a1 , . . . , a2015 is divisible
by 2015. Therefore, if M (k, 2015) = 1, the grasshopper will jump into a hole.
To conclude, the only numbers which are suitable for the terms of the problem are
those which are not relatively prime to 2015.

∗∗∗
16 Mathematical competitions in Croatia in 2015

3.1. Let D be the point on the side BC such that ^DAC = 90◦ . Let us denote
ϕ = ^CDA and x = |CD|. Then cos ^ACB = sin ϕ.

D
ϕ

A B

ϕ
Then we have |AC| = x sin ϕ and |BD| = |AD| = x cos ϕ. We also have ^BAD = 2
and |AB| = 2x cos ϕ cos ϕ2 .
Since |BC| + |AC| = 2|AB|, we get
ϕ
1 + cos ϕ + sin ϕ = 4 cos ϕ cos .
2

By squaring both sides of the equation we get


ϕ
1 + cos2 ϕ + sin2 ϕ + 2 cos ϕ + 2 sin ϕ + 2 sin ϕ cos ϕ = 16 cos2 ϕ cos2 ,
2
and further on

2(1 + cos ϕ)(1 + sin ϕ) = 8 cos2 ϕ(1 + cos ϕ),


1 + sin ϕ = 4(1 − sin2 ϕ),
(4 sin ϕ − 3)(sin ϕ + 1) = 0,
3
sin ϕ = ,
4
where we used cos ϕ 6= −1 and sin ϕ 6= −1, which is valid because the angle ϕ is acute.
3
Hence, cos ^ACB = .
4

3.2. If we write down the given equation in the form 2m p2 = n5 − 1 and factorise the
right-hand side, we get

2m p2 = (n − 1)(n4 + n3 + n2 + n + 1).

Factor n4 + n3 + n2 + n + 1 is odd, so n − 1 is divisible by 2m .


We immediately see that p is odd.
On the other hand, since n is positive, we clearly have n4 + n3 + n2 + n + 1 > n − 1.
Hence p cannot divide n − 1, because otherwise n − 1 would be at least 2m p, and
n4 + n3 + n2 + n + 1 would be at most p, which is less than 2m p. Hence, we have

2m + 1 = n, p2 = n4 + n3 + n2 + n + 1.
Final round – National competition 17

Let us notice that the second equation is equivalent to the following equations:
n4 + n3 + n2 + n = p2 − 1,
n(n + 1)(n2 + 1) = (p − 1)(p + 1).

By plugging n = 2m + 1 into the last equation we get


(2m + 1)(2m + 2)(22m + 2m+1 + 2) = (p − 1)(p + 1),
which leads us to
4(2m−1 + 1)(2m + 1)(22m−1 + 2m + 1) = (p − 1)(p + 1).

Since p is odd, the right-hand side is the product of two consecutive even numbers,
so it is divisible by 8. The left-hand side is not divisible by 8, unless m = 1.
It follows that the only solution is (p, m, n) = (11, 1, 3).

3.3. Let G be the set of all cities in the country. We call a pair (A, Z), where A and
Z are disjoint subsets of G good if all cities in the set A can be visited using only bus
such that no city is visited twice and all cities in the set Z can be visited using only
train such that no city is visited twice.
Let (A, Z) be a good pair such that the set A ∪ Z has the maximum number of
elements. If we prove A ∪ Z = G, the statement of the problem holds.
Let us assume the opposite, i.e. there is a city g which isn’t from A nor Z. Without
loss of generality we can assume that A and Z are non-empty, because otherwise we
can transfer any city from a non-empty set to an empty one.
Let n be the number of cities in the set A, and m the number of cities in the set Z. Let
us arrange the cities from A in the series a1 , . . . , an such that every two consecutive
cities in that series are connected by a direct bus line. Also, let us arrange the cities
from Z in the series z1 , . . . , zm such that every two consecutive cities in that series
are connected by a direct train line.
Since we assumed that the pair (A, Z) is maximum, the cities g and a1 have to be
connected by train (otherwise the pair (A ∪ {g}, Z) would be a good pair whose union
would have more elements than A ∪ Z), and g and z1 have to be connected by bus
(otherwise the pair (A, Z ∪ {g}) would be a good pair whose union would have more
elements than A ∪ Z).
The cities a1 and z1 have to be connected by bus or by train.

z1 Z
...
zm

a1 A
...
an
18 Mathematical competitions in Croatia in 2015

If a1 and z1 are connected by bus, let us put A0 = {z1 , g, a1 , . . . , an } and Z 0 =


{z2 , . . . , zm }. Then (A0 , Z 0 ) is a good pair and the number of elements of A0 ∪ Z 0 is
greater than the number of elements of A ∪ Z, which contradicts the assumption.
If a1 and z1 are connected by train, let us put A00 = {a2 , . . . , an } and Z 00 = {a1 , g, z1 ,
z2 , . . . , zm }. Then (A00 , Z 00 ) is a good pair and the number of elements of A00 ∪ Z 00 is
greater than the number of elements of A ∪ Z, which contradicts the assumption.
Since all cases lead to contradiction, we conclude that the assumption was wrong and
that every city is either in the set A or in the set Z.

3.4. Let F 0 be the intersection of the circumcircle of the triangle ABD and the
line BG (different from B).

F = F′

C D E A

The quadrilateral DAF 0 B is cyclic, so we have ^BF 0 D = ^BAD = ^BAC. Since


GE k AB, we have ^BAC = ^GEC. Hence ^GF 0 D = ^GEC, which means that
DEF 0 G is a cyclic quadrilateral.
Therefrom ^AEF 0 = ^DGF 0 = ^DGB. Since the quadrilateral CDBG is cyclic, we
have ^DGB = ^DCB, so we can conclude F 0 E k BC.
Hence, F 0 is the intersection point of the circumcircle of the triangle ABD and the
line parallel to BC through E, which means that F 0 = F . Hence DEF G is a cyclic
quadrilateral, which means that D, E, F and G lie on the same circle.

3.5. Let us first notice


a − bc a + bc − 2bc bc
= =1−2· ,
a + bc a + bc a + bc
b − ca ca c − ab ab
=1−2· , =1−2· .
b + ca b + ca c + ab c + ab

Now, from a + b + c > 1 it follows that


bc bc bc
> = ,
a + bc a(a + b + c) + bc (a + b)(a + c)
Final round – National competition 19

and
ca ca ab ab
> , >
b + ca (b + c)(b + a) c + ab (c + a)(c + b)
analogously, so we have
a − bc b − ca c − ab 2bc 2ca 2ab
+ + 63− − − .
a + bc b + ca c + ab (a + b)(a + c) (b + c)(b + a) (c + a)(c + b)
Hence, in order to prove the desired inequality, it is enough to prove
2bc 2ca 2ab 3
+ + > .
(a + b)(a + c) (b + c)(b + a) (c + a)(c + b) 2

Multiplying by 2(a + b)(b + c)(c + a) we see that the inequality above is equivalent to

4bc(b + c) + 4ca(c + a) + 4ab(a + b) > 3(a + b)(b + c)(c + a),

i.e.
a2 b + ab2 + b2 c + bc2 + c2 a + ca2 > 6abc.

The last inequality obviously holds since by the AM–GM inequality



6
a2 b + ab2 + b2 c + bc2 + c2 a + ca2 > 6 a6 b6 c6 = 6abc

holds.

∗∗∗

4.1. By plugging x = y = 0 into the given equation we get

f (0)2 = 0, i.e. f (0) = 0.

By plugging x = y = 1 into the given equation we get

f (1)(1 + f (1)) = 2f (1), i.e. f (1)2 = f (1).

We distinguish two cases: f (1) = 0 or f (1) = 1.


If f (1) = 0, by plugging x = 1 into the starting equation we get that for every y ∈ R

f (y)(1 + f (y)) = f (y), i.e. f (y) = 0.

If f (1) = 1, by plugging y = 1 into the starting equation we get that for every x ∈ R

f (x)(x + 1) = x2 + f (x), i.e. xf (x) = x2 .

For x 6= 0 dividing by x we get f (x) = x. Since f (0) = 0, it follows that f (x) = x for
every x ∈ R.
Now we easily check that the functions f (x) = 0 and f (x) = x really satisfy the
starting equation. Thus they are the only solutions.
20 Mathematical competitions in Croatia in 2015

4.2. Let us denote by T the centroid of the triangle ABC, by a, b and c the lengths
of the sides BC, CA and AB respectively and by v the length of the altitude to the
side AB. Let us denote α = ^BAC and β = ^CBA.
Since T is the centroid of the triangle ABC, it follows that
1 1 1 1 1
|T A0 | = |B 0 C| = |CA| = b, |T B 0 | = |A0 C| = |BC| = a, |T C 0 | = v.
3 3 3 3 3

C′

A′
T

C B′ A

We have

P (A0 B 0 C 0 )
= P (A0 B 0 T ) + P (B 0 C 0 T ) + P (C 0 A0 T )
1
= (|T A0 | · |T B 0 | + |T B 0 | · |T C 0 | · sin(π − α) + |T C 0 | · |T A0 | · sin(π − β))
2
1
= (ab + av sin α + bv sin β).
18
Since v = a sin β, v = b sin α, a = c sin α, b = c sin β and c2 = a2 + b2 hold, we get
1
P (A0 B 0 C 0 ) = (ab + a2 sin α sin β + b2 sin α sin β)
18
1 1
= (ab + c2 sin α sin β) = (ab + ab)
18 18
1 2
= ab = P (ABC).
9 9

Hence P (A0 B 0 C 0 ) : P (ABC) = 2 : 9.

4.3. Let us put n = ad. The condition dn + 1 | d2 + n2 can be written as ad2 + 1 |


d2 + a2 d2 .
Then ad2 + 1 divides d2 + a2 d2 − a · (ad2 + 1) = d2 − a as well.
Let us consider all possible signs of the number d2 − a.
If d2 − a > 0, then it must be d2 − a > ad2 + 1 (multiple is greater than or equal to
divisor). Since a is positive, we have d2 > ad2 + a + 1 > d2 , which is impossible.
Final round – National competition 21

If d2 − a < 0, then a − d2 > 0. Then it must be a − d2 > ad2 + 1. Since d is positive,


we have a > ad2 + 1 + d2 > a, which is also impossible.
Hence, the only possibility is d2 − a = 0, which gives a = d2 and n = d3 . In that case
d4 + 1 divides d2 + d6 = d2 (d4 + 1), so all possible numbers n are cubes of positive
integers.

4.4. Since
n(n + 3)
nx = x + · · · + x and = (1 + · · · + n) + n,
| {z } 2
n

the given equation is equivalent to


n n n
X (k + 1)2 X X
+ nx2 − x− k − n = 0,
x+k
k=1 k=1 k=1

i.e.
n ï
(k + 1)2
X ò
− (x + k) + nx2 − n = 0.
x+k
k=1

After some manipulation we get

(k + 1)2 (k + 1)2 − (x + k)2 k+1


Å ã
− (x + k) = = (1 − x) 1 + ,
x+k x+k x+k
so the given equation is equivalent to
n
!
X k+1
(1 − x) n + + n(x2 − 1) = 0,
x+k
k=1

i.e.
2 3 n+1
ï Å ã ò
(1 − x) n + + + ··· + − n(1 + x) = 0.
x+1 x+2 x+n
For x = 1 the equality is obviously satisfied. For x 6= 1 we have

2 3 n+1
Å ã
n+ + + ··· + = n(1 + x),
x+1 x+2 x+n
i.e.
2 3 n+1
+ + ··· + = nx.
x+1 x+2 x+n
If 0 < x < 1, then each of n fractions on the left-hand side is greater than 1, so the
left-hand side is greater than n, and the right-hand side is less than n.
If x > 1, exactly the opposite holds: the left-hand side is less than n, and the right-
hand side is greater than n.
Hence, the only solution is x = 1.
22 Mathematical competitions in Croatia in 2015

4.5. Let us divide the board into 16 2 × 2 squares as in the picture.

In each of those squares at least two cells have to be covered, otherwise we could place
a tromino-tile on three uncovered cells. Hence, at least 32 cells have to be covered,
and we need at least 11 tromino-tiles to do that covering.
The following construction shows that 11 tromino-tiles are enough to satisfy conditions
of the problem.
I.4 Croatian Mathematical Olympiad

Day 1

A1. Let m be a positive integer. We have 2m sheets of paper, with the number 1
written on each of them. We perform the following operation. In every step we
choose two distinct sheets; if the numbers on the two sheets are a and b, then
we erase these numbers and write the number a + b on both sheets. Prove that
after m2m−1 steps, the sum of the numbers on all the sheets is at least 4m .
(IMO Shortlist 2014)

C1. A magical triangulation is a partition of a triangle on smaller triangles by a


finite number of segments whose endpoints are vertices of the triangle or points
in its interior, such that in every point (including the vertices of the triangle)
meets the same number of segments.
What is the maximum number of smaller triangles on which we can divide the
triangle in a magical triangulation? (Counting and Configurations)

G1. Circles k1 and k2 meet at points M and N . The line l meets the circle k1 in
points A and C, and the circle k2 in points B and D so that the points A, B,
C and D are on the line l in that order. Let X be a point on the line M N
such that the point M is between points X and N . Let P be the intersection
of lines AX and BM , and Q the intersection of lines DX and CM .
If K is the midpoint of the segment AD, and L the midpoint of the segment BC,
prove that the lines XK and M L meet on the line P Q. (Matija Bucić)
ú ü
2k
N1. For a positive integer k, let ak = . Prove that the sequence (ak ) contains
k
infinitely many odd numbers.
(bxc denotes the largest integer not greater than x.) (IMO Shortlist 2014)

∗∗∗

Day 2

A2. For all positive real numbers x, y and z prove the following inequality
x2 y2 z2 (x + y + z)3
+ + > .
xy + z yz + x zx + y 3[x (y + 1) + y 2 (z + 1) + z 2 (x + 1)]
2

(Tonći Kokan)

23
24 Mathematical competitions in Croatia in 2015

C2. Let N be a positive integer. In each square of a N × N array initially there is


a zero. In each move it is allowed to choose a row or a column, erase all the
numbers in it and then write numbers from 1 to N in arbitrary order. What is
the maximum possible value of the sum of all numbers in the array?
(Russia 2007)

G2. In an acute triangle ABC we have |AB| > |BC|, and the points A1 and C1
are the feet of altitudes from the vertices A and C, respectively. Let D be the
second intersection of the circumcircles of triangles ABC and A1 BC1 (different
from B). Let Z be the intersection of the tangents of the circumcircle of the
triangle ABC at points A and C, and let lines ZA and A1 C1 meet at the
point X, and lines ZC and A1 C1 at the point Y .
Prove that the point D lies on the circumcircle of the triangle XY Z.
(Romania 2014)

N2. Let n > 2 be a positive integer and p a prime number. If the number p − 1 is
divisible by n, and the number n3 − 1 is divisible by p, prove that 4p − 3 is a
square of an integer.

∗∗∗

Final test for MEMO team selection

A3. Determine all functions f : R → R such that

f (xf (x) + f (xy)) = f (x2 ) + yf (x)

holds for all real numbers x and y. (Tonći Kokan)

C3. Let n > 3 be a positive integer. Some diagonals are drawn in a convex n-gon.
We say that a drawn diagonal is good if it intersects another drawn diagonal in
its interior. Determine the maximum possible number of good diagonals.
(Russia 2014)

G3. Let I be the incentre of the triangle ABC and let D be the point on side AC such
that |AB| = |DB|. Incircle of the triangle BCD touches the lines AC and BD
in points E and F , respectively. Prove that the line EF passes through the
midpoint of the segment DI. (Geometry from the Land of the Incas)

N3. Determine all positive integers x and y such that

x(x2 + 19) = y(y 2 − 10). (Tonći Kokan)

∗∗∗
Croatian Mathematical Olympiad 25

Final test for IMO team selection

A4. Determine all functions f : R → R such that

f (f (x))(x − f (y)) + 2xy = f (x)f (x + y)

holds for all real numbers x and y. (Tonći Kokan)

C4. In a country there are N cities and among some of them there are (two-way)
flight connections. Each flight connects exactly two cities. No city is not directly
connected to every other city. For any two cities A and B there is exactly one
way to fly from A to B using at most two flights.
Prove that N − 1 is a square of a positive integer. (Belarus 2014)

G4. In a quadrilateral ABCD we have ^BAD = 110◦ , ^CBA = 50◦ , ^DCB = 70◦ .
Let M and N be the midpoints of segments AB and CD respectively. Let P
be a point on the segment M N such that |AM | : |CN | = |M P | : |N P | and
|AP | = |CP |. Determine ^CP A. (Japan)

N4. Prove that for every positive integer n there exist integers a and b such that n
divides 4a2 + 9b2 − 1. (Math prize for girls olympiad 2010)

I.4.1 Solutions

Day 1

A1. Let Pk denote the product of all numbers on the sheets after k steps. In each
step we take two sheets with numbers a and b and replace them by sheets with a + b
and a + b, so for the products after k steps and after k + 1 steps we have
Pk
Pk+1 = · (a + b)2 .
ab
It follows that
Pk+1 (a + b)2
= > 4.
Pk ab
m−1
Since P0 = 1, we have Pk > 4k and in particular Pm2m−1 > 4m2 . By the AM–GM
inequality applied to the 2m numbers written on the sheets after m2m−1 steps we
have m−1  1
m
p m
Sm2m−1 > 2m · 2 Pm2m−1 > 2m · 4m2 2m
= 2m · 4 2 = 4m .

C1. Let n be the number of smaller triangles, t the number of points in the trian-
gulation (including the vertices of the triangle), d the number of segments (including
the sides of the triangle) and k the number of segments meeting in each point of the
triangulation.
26 Mathematical competitions in Croatia in 2015

Obviously t · k = 2 · d holds. Furthermore, d segments are sides of n + 1 triangles, so


2d = 3(n + 1) since each segment is a side of exactly two triangles.
Finally, let us consider the sum of inner angles of smaller triangles. That sum is
equal to n · 180◦ . On the other hand, in each of t − 3 points in the interior of the
triangle that sum is equal to 360◦ , so when we add the angles of the big triangle we
get that the sum of angles of the smaller triangles is equal 180◦ + (t − 3) · 360◦ . Hence
n · 180◦ = 180◦ + (t − 3) · 360◦ , i.e. 2t = n + 5. From these equations, it follows that

5k − 6 24
n= = + 5.
6−k 6−k

So, 6 − k divides 24 and the only possibilities for a positive integer n are obtained
if k ∈ {2, 3, 4, 5}, i.e. n ∈ {1, 3, 7, 19}. The maximum possible number of smaller
triangles is 19 and the following example shows that this can be achieved.

G1. Let Y be the second intersection of the circle k1 with the line AX and let Z be
the second intersection of the circle k2 with the line DX.
Since lines AY , DZ and M N pass through the point X, by the converse of the radical
centre theorem, quadrilateral AY ZD is cyclic. Hence ^Y ZX = ^XAD.
Since AY M C is a cyclic quadrilateral, it holds ^Y M Q = ^Y AC = ^XAD, so
^Y M Q = ^Y ZX = ^Y ZQ and it follows that the quadrilateral QY M Z is cyclic.
Analogously, quadrilateral P Y M Z is cyclic, so points P , Q, Z, Y and M lie on the
same circle. From this it follows that ^Y P Q = ^Y ZQ = ^XAD, so P Q k AD.
Hence the quadrilaterals ADP Q and BCP Q are trapezia, and since the midpoints
of bases of a trapezium and the intersection of its diagonals lie on the same line, we
conclude the lines KX and LM pass through the midpoint of the segment P Q. This
finishes the proof.
Croatian Mathematical Olympiad 27

k1
k2

D
C l
K L
B
A

M Z
Y

N1. Let k = 3 · 4l for an arbitrary positive integer l. Then we have


ú ü ú l ü ú l ü ú l ü
2k 23·4 23·4 −2l 23·4 −2l − 1 1
ak = = = = + .
k 3 · 4l 3 3 3

l
Since the positive integer 3 · 4l − 2l is even, we have 23·4 −2l ≡ 4 ≡ 1 (mod 3), so the
l
odd positive integer 23·4 −2l − 1 is divisible by 3.
Therefore, ú ü ú l ü l
2k 23·4 −2l − 1 1 23·4 −2l − 1
ak = = + =
k 3 3 3
is odd, which finishes the proof.

∗∗∗

Day 2

A2. By the CSB inequality we have

x2 y2 z2
Å ã
+ + [x(xy + z) + y(yz + x) + z(zx + y)]
xy + z yz + x zx + y
√ √ √
> (x x + y y + z z)2 . (1)
28 Mathematical competitions in Croatia in 2015

By the inequality between power means we have


Ç √ √ √ å2
x x+y y+z z 3 x+y+z
> ,
3 3
i.e.
√ √ √ (x + y + z)3
(x x + y y + z z)2 > . (2)
3
By the AM–GM inequality we have
x2 + y 2 y2 + z2 z 2 + x2
x2 + y 2 + z 2 = + + > xy + yz + zx. (3)
2 2 2
So, from (1), (2) and (3) it follows that
√ √ √
x2 y2 z2 (x x + y y + z z)2
+ + > 2
xy + z yz + x zx + y x y + y 2 z + z 2 x + zx + xy + yz
(x + y + z)3
>
3(x2 y + y 2 z + z 2 x + x2 + y 2 + z 2 )
(x + y + z)3
= .
3[x2 (y + 1) + y 2 (z + 1) + z 2 (x + 1)]

C2. Let us first prove the following statement:


Let k a positive integer. In an array with dimensions m × n (m, n > k) all squares
are initially white. In each move it is allowed to choose a row or a column and colour
the squares in it so that there is at most k black squares (other squares are white).
After each move there will be at least (m − k) · (n − k) white squares in the array.
We will prove the above statement by induction on the number of squares in an
array. For m = k or n = k the statement obviously holds. Let us assume that
m, n > k and that the statement holds for all arrays of dimensions smaller than
m × n. Without loss of generality we may assume that in the last move we have
performed the transformation on some row. In that row there must me at least n − k
white squares. If we disregard that row we may assume that all previous moves were
made on an array of dimensions (m − 1) × n. By the inductive hypothesis the smaller
array has at least (m − 1 − k) · (n − k) white square, and this implies that the array
with dimensions m × n has at least (n − k) + (m − 1 − k)(n − k) = (m − k)(n − k)
white squares. This proves the statement.
Let m = n = N and let us assume that after some number of moves number i
appears Ai times in the array (1 6 i 6 N ). If we consider the squares in which there
is number N to be black, and all other squares to be white, then the above statement
implies
AN 6 N 2 − (N − 1)2 .
More generally, if we consider the squares in which numbers S + 1, . . . , N + 1, N
(S = N − 1, N − 2, . . . , 0) to be black, and all other squares to be white, then the
above statement (with k = N − S) implies
AS+1 + · · · + AN 6 N 2 − S 2 .
Croatian Mathematical Olympiad 29

If we sum all inequalities for S = N − 1, N − 2, . . . , 0 we get

N · AN + (N − 1)AN −1 + (N − 2)AN −2 + · · · + 2A2 + A1

6 N · N 2 − ((N − 1)2 + (N − 2)2 + · · · + 02 )

(N − 1)N (2N − 1) 4N 3 + 3N 2 − N
= N3 − = .
6 6

On the other hand, we can achieve the equality in the following way. We apply the
transformation by alternating between rows and columns, filling the rows from top
to bottom and the columns from left to right. We write the numbers in each row in
a descending order from left to right and in each column from top to bottom. After
these transformations in the first row and the first column there will be number N ,
in the squares of the second row and the second column which are not in the first
row/column there will be number N − 1, and so on.

G2. Let α = ^BAC, β = ^CBA and γ = ^ACB.

B
D

X
C1

A1

C A

Since ^CAZ = ^ZCA = β (angle between the tangent and the chord), it follows
that ^XZY = ^AZC = 180◦ − 2β.
30 Mathematical competitions in Croatia in 2015

Quadrilateral CAC1 A1 is cyclic, so we have ^C1 A1 B = ^Y A1 C = α and ^BC1 A1 =


^AC1 X = γ. Since ^XAC1 = 180◦ − ^CAZ − ^BAC = 180◦ − β − α = γ, from the
isosceles triangle AXC1 we get ^C1 XA = 180◦ − 2γ. Analogously, from the isosceles
triangle A1 Y C we get ^CY A1 = 180◦ − 2α.
Let us show that the quadrilateral A1 AXD is cyclic. Since quadrilaterals A1 C1 BD
and CABD are cyclic, we have ^C1 DB = ^C1 A1 B = α and ^ADB = ^ACB = γ,
so ^ADC1 = ^ADB − ^C1 DB = γ − α. Furthermore, ^A1 DC1 = ^A1 BC1 = β, so

^A1 DA = ^A1 DC1 − ^ADC1 = β − (γ − α) = α + β − γ = 180◦ − 2γ = ^A1 XA,

and the quadrilateral A1 AXD is cyclic.


Similarly, ^CDC1 = ^CDA + ^ADC1 = β + (γ − α) = 180◦ − 2α = ^CY C1 , so the
quadrilateral CC1 DY is cyclic.
Now we have

^Y DX = ^Y DC + ^CDA + ^ADX
= ^Y C1 C + ^CBA + ^AA1 X
= ^A1 C1 C + ^CBA + ^AA1 C1
= (90◦ − γ) + β + (90◦ − α) = 2β,

so ^XZY + ^Y DX = 180◦ , from what we conclude that the quadrilateral XDY Z is


cyclic.

N2. Since n divides p − 1, there exists a positive integer a such that p − 1 = an. We
also have p − 1 > n.
From the condition that n3 −1 = (n−1)(n2 +n+1) is divisible by the prime number p,
it follows that n2 + n + 1 is divisible by p. Indeed, 1 6 n − 1 < n + 1 6 p, so n − 1
cannot be divisible by p.
Hence an + 1 | n2 + n + 1. This implies that 1 6 a 6 n + 1 (because if a > n + 2,
then an + 1 > (n + 2) · n + 1 = n2 + 2n + 1 > n2 + n + 1, which is impossible).
From the same divisibility it follows that an+1 | a·(n2 +n+1)−n·(an+1) = (a−1)n+a,
which is positive, so we must have (a − 1)n + a > an + 1, i.e. a > n + 1.
It follows that a = n + 1 and p = n2 + n + 1.
Hence 4p − 3 = 4n2 + 4n + 1 = (2n + 1)2 , which finishes the proof.

∗∗∗

Final test for MEMO team selection

A3. If we plug x = 0 in the equation, we get f (f (0)) = f (0) + yf (0) for all y ∈ R.
This implies f (0) = 0.
Let f (1) = c and plug in x = 1, it follows that f (c + f (y)) = c(1 + y) for all y ∈ R.
Croatian Mathematical Olympiad 31

If c = 0, then f (f (y)) = 0 for all y ∈ R. By applying f to the given equation, it follows


that 0 = f (f (x2 ) + yf (x)). Let us assume that f (t) 6= 0 for some t ∈ R. Then we can
find y ∈ R such that t = f (t2 ) + yf (t) and we get 0 = f (f (t2 ) + yf (t)) = f (t) 6= 0,
which is a contradiction. Hence, in this case f (x) = 0 for all x ∈ R.
If c 6= 0, then it follows that f is injective. If we plug in y = 0, we get f (xf (x)) =
f (x2 ), i.e. f (x) = x for all x 6= 0. As we know f (0) = 0, we conclude that f (x) = x
is another solution.
It is easy to check that f (x) = 0 for all x ∈ R and f (x) = x for all x ∈ R are indeed
solutions.

C3. Let M (n) be the maximum possible number of good diagonals in a convex n-gon.
We will show that M (n) = n − 2 if n is even and M (n) = n − 3 if n is odd. For
any n, we can draw all n − 3 diagonals from one vertex A and one more diagonal
joining two vertices adjacent to A if n > 3. We show by mathematical induction that
M (n) > n − 2 if n is even. For n = 4 the claim is true as both diagonals in a convex
quadrilateral are good. Let us assume that the claim is true for n − 2 and consider
a convex n-gon A1 A2 . . . An . We draw diagonals An−2 An , A1 An−1 and we use the
assumption on A1 A2 . . . An−2 , so M (n) > M (n − 2) + 2 > n − 4 + 2 = n − 2.
We also prove by induction that M (n) 6 n − 2 if n is even and that M (n) 6 n − 3
if n is odd. Obviously, M (3) = 0 and M (4) = 2. Let us assume that the claim holds
for all n smaller than k.
Let us consider a choice of diagonals of a convex k-gon for which M (k) is achieved.
First case: if there are two good diagonals that intersect, they divide the k-gon into 4
part, each having ai > 0 vertices, not counting the endpoints of these diagonals (for
1 6 i 6 4). Since there is no other segment that intersects these two diagonals, we
have
4
X 4
X
M (k) = 2 + M (ai + 2) 6 2 + (ai + 2 − 2) = 2 + (k − 4) = k − 2.
i=1 i=1

If k is odd, then at least one of the numbers ai must also be odd, so we get the bound
k − 3 in this case.
Second case: if there are no two good diagonals that intersect, then there is automat-
ically at most k − 3 good diagonals as that is the maximum number of diagonals that
we can draw from one vertex.

G3. We denote by a, b and c the lengths of the sides BC, CA and AB respectively.
Let K be the point at which the incircle of the triangle ABC touches the line AC.
Let N be the foot of the altitude from B in triangle ABC and let J be the intersection
of line EF and the altitude BN .
Note that ^JN E = ^AKI = 90◦ and
1 ^ADB ^BAC
^N EJ = (180◦ − ^F DE) = = = ^IAK.
2 2 2
32 Mathematical competitions in Croatia in 2015

I J

C E D K N A

c+d−a
Let d = |CD|. Then |ED| = and
2
c+d−a b−d b+c−a
|EN | = |ED| + |DN | = + = = |AK|.
2 2 2
Hence the triangles AIK and EJN are congruent. It follows that |IK| = |JN |, so
|IJ| = |KN |. Furthermore,
b+c−a b−d c+d−a
|IJ| = |KN | = |AK| − |AN | = − = = |ED|.
2 2 2
From this we conclude that EDJI is a parallelogram and hence its diagonals have
the same midpoint.

N3. Let d be the greatest common divisor of x and y, and let us denote x = da
and y = db, where a and b are relatively prime positive integers. Then the equation
becomes
a(d2 a2 + 19) = b(d2 b2 − 10),
what can be rewritten as
19a + 10b = d2 (b3 − a3 ) = d2 (b − a)(b2 + ba + a2 ).
As positive integer b − a divides the right-hand side, it must also divide 19a + 10b =
10(b − a) + 29a. This implies that b − a divides 29a. As a and b are relatively prime,
so are b − a and a. We conclude that b − a divides 29, so b − a = 29 or b − a = 1.
If b − a = 29, the equation becomes a + 10 = d2 (3a2 + 87a + 841), which has no
solution since the left-hand side is obviously less than the right-hand side.
If b − a = 1, the equation becomes
29a + 10 = d2 (3a2 + 3a + 1).
If d > 3, then the left-hand side is less than the right-hand side. If d = 2, then we get
the quadratic equation 12a2 − 17a − 6 = 0, which has no integral solution. If d = 1,
we get 3a2 − 26a − 9 = 0, which has one integral solution a = 9.
Hence, the only solution is (x, y) = (9, 10).

∗∗∗
Croatian Mathematical Olympiad 33

Final test for IMO team selection

A4. Let us denote a = f (0). If we put x = y = 0 in the given equation, we get

f (a)(−a) = a2 , (4)

which implies that a = 0 or a = −f (a).


If we put x = a, y = 0 in the given equation, we get f 2 (a) = 0, which, in combination
with (4), gives a = 0, i.e. f (0) = 0.
Putting y = 0 in the given equation gives us

xf (f (x)) = f 2 (x). (5)

Let us denote b = f (1). Putting x = 1 into (5) gives us

f (b) = b2 . (6)

Putting x = 1, y = b into the given equation and using (6) gives us

b2 (1 − b2 ) + 2b = bf (b + 1). (7)

Similarly, putting x = b, y = 1 into the given equation and also using (6) gives us

2b = b2 f (b + 1). (8)

If b = 0, putting x = 1 into the given equation gives us 2y = 0, which is clearly not


possible for all y ∈ R. Thus b 6= 0 and (8) gives us

bf (b + 1) = 2. (9)

If we combine (7) and (9) we get

b2 (1 − b2 ) + 2b = 2,
(b − 1)2 (b2 + 2b + 2) = 0.

The only real number that satisfies this equation is b = 1. Therefore, f (1) = 1.
Putting x = 1, y = x into the given equation gives us

f (x + 1) + f (x) = 2x + 1. (10)

By putting y = 1 into the given equation and using (5), we get

f 2 (x)(x − 1) + 2x2 = xf (x)f (x + 1). (11)

For a given x, let t = f (x). Combining (10) and (11) we get

t2 (x − 1) + 2x2 = xt(2x + 1 − t),


(t − x)(2tx − t − 2x) = 0.
34 Mathematical competitions in Croatia in 2015

Thus, for every x ∈ R it holds

2x
f (x) = x or f (x) = .
2x − 1

Let S = {x ∈ R | f (x) 6= x}. From (10) we conclude that x ∈ S =⇒ x + 1 ∈ S.


6 ∅. If we take an
Therefore, S is either empty or infinite. Let us assume that S =
arbitrary x ∈ S we have

2x 2x + 2
f (x) = and f (x + 1) = . (12)
2x − 1 2x + 1

Combining (10) and (12) we get

2x + 2 2x
+ = 2x + 1,
2x + 1 2x − 1
8x3 − 4x2 − 6x + 1 = 0.

This equation has at most three distinct solutions which contradicts the assumption
that S is an infinite set. Therefore, S is an empty set and f (x) = x, for all x ∈ R.
An easy check verifies that this is indeed a solution.

C4. Let a be an arbitrary city and let it be connected to cities x1 , . . . , xk . We denote


by Bi the set of all cities connected to xi that are different from a, for i = 1, . . . , k.
Sets B1 , . . . , Bk are pairwise disjoint because there is exactly one way to fly from xi
to xj in at most two flights.
If we consider two indices i and j and an arbitrary city y ∈ Bi , there exists a unique
city z ∈ Bj such that y and z are connected (because there must be a way to come
from y to xj in at most two flights and there is no direct flight).
We conclude that every element of some set Bi has exactly k connections and that
all sets Bi have the same number of elements. Let m be the number of elements of
the sets Bi . Then N = 1 + k + km.
Furthermore, let us consider some city b ∈ B1 . It is connected to x1 and also to exactly
one city ci ∈ Bi for i = 2, . . . , k. Note that xi is connected to ci for i = 2, . . . , k. By
the same way as above (with b instead of a) we conclude that cities xi (for i = 2, . . . , k)
have k connections. But we already know that cities xi have m + 1 connections (one
to the city a and m to cities in Bi ). Hence k = m + 1 and N − 1 = k + km = k 2 .

G4. Let K be the intersection of lines AD and BC. Since ABCD is a cyclic quadri-
lateral, triangles AKB and CKD are similar. As M and N are midpoints of respective
segments, we have ^AKM = ^CKN . It follows that triangles AKM and CKN are
similar, too.
Croatian Mathematical Olympiad 35

A′ N

D P

A M B

Hence |KM | : |KN | = |AM | : |CN |. By the condition in the problem, we have
|KM | : |KN | = |M P | : |N P |. We conclude that KP is the angle bisector of the
angle ^N KM , which implies that KP is the angle bisector of the angle ^AKC.
We will show that the quadrilateral AP CK is cyclic. Indeed, let point A0 6= A be on
the line KA such that |A0 P | = |AP | = |CP |. Then the triangles KA0 P and KCP
are congruent, and the quadrilateral A0 P CK is deltoid. Since the triangle A0 AP is
isosceles, we have ^P AK = ^P AA0 = ^AA0 P = 180◦ − ^P A0 K = 180◦ − ^KCP .
Hence, AP CK is a cyclic quadrilateral.
Since ^AKC = 180◦ − ^BAD − ^CBA = 20◦ , it follows that ^CP A = 160◦ .

N4. If n is odd, let n = 2k + 1 for some non-negative integer k. For a = k and b = 0


we have 4a2 + 9b2 − 1 = (2k + 1)(2k − 1), so n divides 4a2 + 9b2 − 1.
If n is not divisible by 3, let n = 3k+r for some non-negative integer k and r ∈ {1, −1}.
For a = 0 and b = k we have 4a2 +9b2 −1 = (3k +1)(3k −1), so n divides 4a2 +9b2 −1.
If n is divisible by 6, let n = 2r 3s m for some positive integers r, s and m such that m
is relatively prime to 6. Since 2r and 3s m are relatively prime, there exist non-zero
integers k and l such that 2r k + 3s ml = 1. Squaring this equation we get
22r k 2 + 32s m2 l2 + 2 · 2r 3s mkl = 1,
i.e. −2nkl = 22r k 2 +32s m2 l2 −1. For a = 2r−1 k and b = 3s−1 ml we have 4a2 +9b2 −1 =
22r k 2 + 32s m2 l2 − 1, so n divides 4a2 + 9b2 − 1, which finishes the proof.
Mathematical competitions in Croatia in 2016

In Croatia, children undergo twelve years of formal education – from 1st to 8th grade
in primary school and from 9th to 12th grade in secondary school. Mathematical com-
petitions are held for students from 4th to 12th grade. Competitions at the secondary
school level have two categories – category A (for students who attend mathematical
grammar schools) and category B (for all other secondary school students).
In the year 2016, 17883 students participated in the first round of the competitions,
among them 958 in category A for secondary schools. The City competitions were
held on 21st January. All tests at this level consist of five easier problems and two
harder problems, as well as tests for category B at County competitions. All other
tests at County competitions and all the tests at National competition consist of five
problems.
On County competitions held on 23rd February, 2021 primary school students and
1156 secondary school students (503 of them in category A) participated. The best
90 students from primary school (from 5th to 8th grade), and 164 secondary school
students (91 in category A and 73 in category B) were invited to the National com-
petition in Primošten (from 4th to 6th April).
After the National competition, an additional three-round team selection process,
called Croatian Mathematical Olympiad, was organised. Top 21 students in the Na-
tional competition from category A took two tests on 16th and 17th April. Separate
final selection tests for the IMO and MEMO teams took place on 24th of April. All
tests consisted of four olympiad-style problems from four different areas: algebra,
combinatorics, geometry and number theory.
After the Croatian Mathematical Olympiad, the teams for the 57th International
Mathematical Olympiad (IMO) in Hong Kong and the 10th Middle European Math-
ematical Olympiad (MEMO) in Austria were selected:

IMO MEMO
1
Adrian Beker (grade 11) Luka Banović 3 (grade 10)
Domagoj Bradač 1 (grade 12) Lukas Novak 4 (grade 11)
Andrija Mandić 1 (grade 12) Petar Nizić-Nikolac 1 (grade 10)
Petar Orlić 1 (grade 12) Patrik Papac 5 (grade 11)
Daniel Paleka 2 (grade 12) Ivan Sinčić 3 (grade 10)
Leon Starešinić 1 (grade 12) Tadej Petar Tukara 1 (grade 10)

1
XV. gimnazija, Zagreb, 2 Gimnazija Franje Petrića, Zadar, 3 Gimnazija Andrije
Mohorovičića, Rijeka, 4 Gimnazija Josipa Slavenskog, Čakovec, 5 Gimnazija Dubrovnik,
Dubrovnik.

37
II.1 First round – City competition

1.1. Can the sum of squares of three consecutive integers be divisible by 2016?
(Ilko Brnetić)

1.2. Anja and Vanja participated in a race. Number of participants who ended
the race before Anja equals the number of participants who ended after her.
Number of participants who ended the race before Vanje is three times the
number of participants who ended after her. Exactly 10 participants ended the
race between Anja and Vanja. If all the participants ended the race and no two
of them have the same time, find the total number of participants.
(The Netherlands 2014)

1.3. Let a and b be positive real numbers such that a2 + b2 = 8 and a6 + b6 = 416.
Find ab. (Željko Hanjš)

1.4. Nine congruent rectangles with integer sides are used to cover the rectangle
20 × 9. Find all possible dimensions of the nine rectangles. (Azra Tafro)

1.5. Let ABCD be a square with side length a. Points A and C are centres of two
circles going through B and D. If the circles intersect diagonal AC at points
M and N , find the area of the quadrilateral BM DN . (Mea Bombardelli)

1.6. The lengths of the legs of a right triangle are a and b, and the length of its
hypotenuse is c. If one of the angles measures 75◦ , prove that c2 = 4ab.
(Tamara Srnec)

1.7. On an island there are 20 clubs. Each inhabitant of the island is a member of
one or two clubs. Each club has at most 25 members, and for each pair of clubs
there is an inhabitant which is a member of both clubs. Find the minimum and
maximum possible number of inhabitants of the island. (Argentina 2012)

∗∗∗

2.1. Find all c ∈ R such that one solution of the quadratic equation
27x2 − 12x + c = 0
is the square of the other. (Stipe Vidak)

2.2. Let a = 123456789 and N = a3 − 2a2 − 3a. Prove that N is a multiple of


540. (Matija Bašić)

38
First round – City competition 39

2
2.3. Let w 6= 1 be a complex number such that |w| = 1 and let z = .
1−w
Find Re z. (Stipe Vidak)

2.4. Points A, B, C, D, E lie in that order on the circle with diameter AE.
Determine ^ABC + ^CDE. (Stipe Vidak)

2.5. Some day, eleven friends decided to paint a large fence. Friends agreed that some
of them will start painting at 9 AM and that the last will end at 4 PM. Each of
them started working at the beginning of the hour and worked for exactly two
hours. Can we be sure that in some period at least four friends worked at the
same time? (Nikola Adžaga)

2.6. Let ABCD be a rectangle with centre O and let P and Q be the points on
diagonal AC such that |AP | = |P Q| = |QC|. If line P B meets side AD at M
and line BQ meets side CD at N , prove that areas of the triangles M P O and
N QO are equal. (Mea Bombardelli)

2.7. Let n > 3 be an integer. Square ABCD is divided into n2 rectangles by lines
p1 , . . . , pn−1 parallel to AB and lines q1 , . . . , qn−1 parallel to BC. AB, CD
lie on p0 , pn , respectively, and BC, AD on q0 , qn , respectively. For each i =
1, . . . , n − 1, line pi lies between lines pi−1 and pi+1 , while line qi lies between
lines qi−1 and qi+1 . For 1 6 i, j 6 n, let Ai,j be the rectangle surrounded by
pi−1 , pi , qj−1 and qj .
If the areas of Ai,j and Aj,i are equal for each pair (i, j) (1 6 i < j 6 n), prove
that Ai,i is a square for i = 1, . . . , n. (Mea Bombardelli)

∗∗∗

3.1. Find all pairs (a, b) of positive integers such that 1 < a, b 6 100 and

1 1
+
loga 10 logb 10

is a positive integer. (Stipe Vidak)

3.2. Let ABC be a triangle with centroid T and let D and E be midpoints of BC and
CA, respectively. If AT E is an equilateral triangle, determine cos ^DAB.
(Mathematics Stack Exchange)

3.3. Find all pairs (x, y) of real numbers such that

x4
sin(x2 − 5y) − 1 > . (Tamara Srnec)
2016
40 Mathematical competitions in Croatia in 2016

3.4. Let ABCDA0 B 0 C 0 D0 be a cube with side length a. Let P be the ortho-
gonal projection of B to the diagonal AC 0 . Find the volume of the pyramid
ABCDP . (Stipe Vidak)

3.5. A square is divided into finitely many smaller squares with integer perimeters.
Is the perimeter of the given square necessarily an integer? (Ukraine 2011)

3.6. Determine all values of real parameter a such that the inequality
sin2 x + a cos x + a2 > 1 + cos x
holds for all x ∈ R. (Stipe Vidak)

3.7. Determine all positive integers n such that the number nn − 3 is multiple of
10. (W. Sierpiński, Elementary Theory of Numbers)

∗∗∗

4.1. A sequence (an ) is given: a1 = 1, a2 = 1,


a22 a2 a2
an+1 = + 3 + ··· + n for n ∈ N, n > 2.
a1 a2 an−1
Find a2016 . (Mathematical Reflections 2009)

4.2. Two numbers a and b are chosen at random from the interval [0, 1]. Find the
probability that the equation ax2 + bx + a = 0 has no real solutions.
(Kristina Ana Škreb)

4.3. Let ABCDEF be a hexagon inscribed in a circle. Segment BE meets segments


AC and DF at G and H, respectively. If |CG| = |HG| = 3, |BG| = |HD| = 2
and |HF | = 5, determine |AC|. (Mea Bombardelli)

4.4. Let a, b and c be integers. If 4a+5b−3c is divisible by 19, prove that 6a−2b+5c
is also divisible by 19. (Mea Bombardelli)

4.5. Is it possible to colour each cell of a 8 × 8 table in one of 16 colours so that for
any pair of colours there are two adjacent cells in these colours? (Two cells are
adjacent if they share a common side.) (Ukraine 2012)

4.6. Parabola y = x2 + ax + b meets coordinate axes in three distinct points A, B


and C. Centre of the circumcircle of the triangle ABC lies on the line y = x.
Prove that a + b + 1 = 0. (St. Petersburg County Competition)

4.7. Determine all pairs of positive integers (x, y) such that x2 − y! = 2016.
(Mathematics Stack Exchange)
II.2 Second round – County competition

1.1. The perimeter of a right-angled triangle is 18 and its area is 9. Find the length
of its hypotenuse. (Ilko Brnetić)

1.2. a) Prove that there are no two positive integers such that the difference of
their squares is 987654.
b) Prove that there are no two positive integers such that the difference of
their cubes is 987654. (Ilko Brnetić)

1.3. Let a, b, c be real numbers. Find the minimum possible value of

a2 + 5b2 + 8c2 − 4ab − 4bc − 8c + 24,

and determine a, b and c for which this value is attained. (Ilko Brnetić)

1.4. Let ^BAC = 2^ABC holds in a triangle ABC. The angle bisector of C meets
side AB at point D. Prove that

|BC| = |AD| + |AC|. (Ukraine 2011)

1.5. In how many ways can we colour the unit squares of a 2 × 2016
board in two colours such that no three unit squares of the same
colour can be covered with a single tile shaped as in the following
picture? The tile can be rotated.
(Brazil 2011)

∗∗∗

2.1. A rectangle is inscribed in an isosceles right-angled triangle of two sides each


equal to 10. One side of the rectangle lies on the hypotenuse and the other two
vertices on the shorter sides of the triangle. Find the maximum possible area
of such a rectangle. (Folklore)

2.2. Let a, b and c be the complex solutions of the equation x3 −2x+2 = 0. Determine
a+1 b+1 c+1
+ + . (Folklore)
a−1 b−1 c−1

2.3. How many ordered pairs (m, k) of positive integers satisfy

20m = k(m − 15k) ? (Ilko Brnetić)

41
42 Mathematical competitions in Croatia in 2016

2.4. Let A and B be points on circle k and let P be a point on the shorter arc AB.
¯
Let Q and R be points on k, different from P , such that |AP | = |AQ| and
|BP | = |BR|. Let T be the intersection of lines AR and BQ. Prove that the
lines P T and AB are perpendicular to each other. (Ivan Kokan)

2.5. Unit squares of a 2 × 50 board should be coloured in two colours, red and blue,
such that the following conditions hold:
• there are both colours on the board;
• by removing all red squares, the board remains connected;
• by removing all blue squares, the board remains connected.
The board is connected if one can get from each unit square to every other by
a sequence of adjacent squares. Two squares are adjacent if they share a side.
In how many ways this can be achieved? (Japan 2012)

∗∗∗

3.1. Let x and y be real numbers such that sin x + sin y = 31 . Prove that

26
sin(3x) + sin(3y) 6 . (Matija Bašić)
27

3.2. Determine all pairs (a, b) of positive integers such that

a3 − 3b = 15,
b2 − a = 13. (Tonći Kokan)

3.3. An isosceles triangle ABC (|AB| = |AC|) is inscribed in a circle k. Let D be


the point on the base BC, k1 the circumcircle of a triangle ABD and point E
on circle k1 . Let us assume that the line AE intersects circle k at points A and
F , such that F lies between A and E. If lines DE and BF intersect at point
G, prove that |EG| = |GF |. (Mea Bombardelli)

3.4. Let k be a circle with diameter AB and t line tangent to k at a point A. Let P
be a point on k and N be the orthogonal projection of P onto line t.
Determine ^ABP such that |P B|+|P N | attains the maximum possible value.
(Italy)

3.5. Consider all rectangular boards which can be coloured in such a way that every
row has exactly 14 blue cells, every column has exactly 10 red cells and there
are exactly 3 cells on the board which are neither blue nor red.
Find the dimensions of such a board which has the minimum total number of
cells. (Mathematics Stack Exchange)
Second round – County competition 43

∗∗∗

4.1. Let a0 , a1 , . . . , an be real numbers such that

a0 + a1 x + . . . + an xn = (x + 1)3 (x + 2)3 · · · (x + 672)3 , for all x ∈ R.

Determine the sum


a2 + a4 + a6 + . . . + a2016 . (Japan 2013)

4.2. Prove that, for every positive integer n > 3, there are n different positive integers
whose reciprocals add up to 1. (New Zealand 2013)

4.3. Let ABC be an acute-angled triangle such that |AB| < |AC|. Let point D be
on the side BC. The altitude from B to line AD intersects the circumcircle of
triangle ABD in points B and E. If lines DE and AC are perpendicular to
each other, prove that AD bisects angle ^BAC. (Stipe Vidak)

4.4. Determine all pairs (a, b) of integers such that (7a − b)2 = 2(a − 1)b2 .
(Mathematics Stack Exchange)

4.5. Let n be a positive integer. In how many ways can we fill n × n board with
numbers 1, 2, −1, −2 so that the product of numbers in each row is −2, and the
product of numbers in each column is also −2? (Thailand 2015)
II.3 Final round – National competition

1.1. Determine the sum


22 + 1 32 + 1 1002 + 1
+ + · · · + .
22 − 1 32 − 1 1002 − 1
(Ratko Višak)

1.2. A segment AD of length 3 is given. Let B and C (C 6= A) be points on the


circle with diameter AD such that |AB| = |BC| = 1. Determine |CD|.
(Hong Kong 1992)

1.3. Determine all triples (x, y, z) of real numbers such that


1 1 1 1 1 1 1 1 1
+ = , + = , + =
x y+z 3 y z+x 5 z x+y 7
hold. (Hong Kong 2000)

1.4. Let a, b and c be positive integers such that


b 1
c=a+ − .
a b
Prove that c is the square of an integer. (New Zealand 2013)

1.5. There are 15 points marked in the plane. Some of them are coloured red, some
blue, and the rest are coloured green. It is known that the number of the red
points is greater than the number of the blue points and greater than the number
of the green points. The sum of lengths of all segments whose one endpoint is
red and the other one is green is 31. The sum of lengths of all segments whose
one endpoint is green and the other one is blue is 25. The sum of lengths of all
segments whose one endpoint is blue and the other one is red is 5.
Determine the number of points of each colour. (Belarus 2010)

∗∗∗

2.1. Let a, b and c be real numbers such that a + 2b + c > 0 and a − 2b + c < 0.
Prove b2 > ac. (Petar Bakić)

2.2. Determine all pairs (m, n) of positive integers for which exist integers a, b and
c that satisfy
a + b + c = 0 and a2 + b2 + c2 = 2m · 3n . (Ilko Brnetić)

44
Final round – National competition 45

2.3. Let ABC be a triangle such that |AB| > |AC|. Let t be the tangent to the
circumcircle of the triangle ABC at the point A. The circle centred at A that
passes through C intersects the segment AB in the point D, and the line t in
the points E and F such that C and E are on the same side of the line AB.
Prove that the incentre of the triangle ABC lies on the line DE. (Miljen Mikić)

2.4. Determine all triples (x, y, z) of positive real numbers that satisfy
1 1 1
x3 + 2y 2 + = 1, y 3 + 2z 2 + = 1, z 3 + 2x2 + = 1.
4z 4x 4y
(Stipe Vidak)

2.5. A board with 2016 rows and 2017 columns is given. Is it possible to remove two
cells from the last column of the board so that the obtained board can be tiled
using only tiles in the picture? The rotation of the tiles is allowed.

(Nikola Adžaga)

∗∗∗

3.1. In a convex quadrilateral ABCD, |AD| = |CD| and ^ADC = 90◦ . If |AB| = a,
|BC| = b, |BD| = d, ^ABC = β, prove that

2d2 = a2 + b2 + 2ab sin β. (Belarus 2008)

3.2. Prove that there does not exist a positive integer k such that

k + 4 and k 2 + 5k + 2

are cubes of positive integers. (Miljen Mikić)

3.3. Let x, y and z be positive real numbers such that xyz = 1. Prove the inequality

x6 + 2 y 6 + 2 z 6 + 2 x y z
Å ã
+ + > 3 + + . (Tonći Kokan)
x3 y3 z3 y z x

3.4. Let H be the orthocentre of the acute triangle ABC. The circle circumscribed
to ABH has centre S and intersects the segment BC at points B and D. Let P
be the intersection of line DH and segment AC, and let Q be the circumcentre
of the triangle ADP .
Prove that the quadrilateral BDQS is cyclic. (Korea 2015)
46 Mathematical competitions in Croatia in 2016

3.5. A board with 2016 rows and 2017 columns is given. Is it possible to remove two
cells from the last column of the board so that the obtained board can be tiled
using only tiles in the picture? The rotation of the tiles is allowed.

(Nikola Adžaga)

∗∗∗

4.1. Find all functions f : R → R such that

f (xy + 1) = f (x)f (y) − f (y) − x + 2

holds for all x, y ∈ R. (China 2003)

4.2. Numbers 1, 2, . . . , 2016 are written in that order in a single line. In each
following line the sums of consecutive numbers from the previous line are written
in order. For example, in the second line the numbers 3, 5, . . . , 4031 are written.
The last line has only one number. Which number is that? (Mea Bombardelli)

4.3. In a convex quadrilateral ABCD we have

^BAC = 48◦ , ^CAD = 66◦ , ^CBD = ^DBA.

Determine the angle ^BDC. (Stipe Vidak)

4.4. Determine all triples (m, n, k) of positive integers such that 3m + 7n = k 2 .


(Ilko Brnetić)

4.5. There are 200 cyclists in a race. At the beginning of the race, the cyclists are
lined up one behind the other. We say that a cyclist overtakes if he changes
positions with the cyclist immediately in front of him. During the race the order
of the cyclists changes only if one of them overtakes.
Let A be the number of all possible orders at the end of a race in which each
cyclist overtook exactly once, and let B be the number of all possible orders at
the end of a race in which each cyclist overtook at most once. Prove that

2A = B. (Iran 2013)
Final round – National competition 47

II.3.1 Solutions

1.1. Let us denote


22 + 1 32 + 1 1002 + 1
S= + 2 + ··· + .
2 −1 3 −1
2 1002 − 1

Let us write the sum S in different form:


22 − 1 + 2 32 − 1 + 2 1002 − 1 + 2
S= + + · · · +
22 − 1 32 − 1 1002 − 1
2 2 2
=1+ 2 +1+ 2 + ··· + 1 +
2 −1 3 −1 100 − 1
2

2 2 2
= 99 + 2 + + ··· + .
2 − 1 32 − 1 1002 − 1

2 2 1 1
Since = = − for every k > 2, we get
k2 − 1 (k − 1)(k + 1) k−1 k+1
2 2 2 2
S = 99 + + 2 + ··· + 2 +
−1 3 −1
22 99 − 1 100 − 1
2

1 1 1 1 1 1 1 1
= 99 + − + − + · · · + − + −
1 3 2 4 98 100 99 101
1 1 1
= 99 + 1 + − −
2 100 101
1014849
= .
10100

1.2. Let S be the centre of the given circle and let T be the intersection of the
segments SB and AC.

A S D

Tha angle ^ACD is right by Thales’ theorem.


We have ^ASB = ^CSB, which implies ^ABS = ^CBS. By S–A–S theorem the
triangles ABT and CBT are congruent. In particular, we have ^AT B = ^CT B, so
we get ^AT B = 90◦ , i.e. AT ⊥ BS. In other words, AT is an altitude of the triangle
ASB.
48 Mathematical competitions in Croatia in 2016

Let va be length of the altitude of the triangle ASB from vertex S. Since |AB| = 1,
|AS| = 23 and triangle ASB is isosceles, we get
 Å ã Å ã2 √
3 2 1
va = − = 2.
2 2

Length of altitude AT can be determined by expressing the area of the triangle ASB
in two ways:
|BS| · |AT | |AB| · va
= .
2 2
√ √
2 2 4 2
From this we get |AT | = 3 , i.e. |AC| = 2|AT | = 3 and finally, by Pythagorean
theorem,
» 7
|CD| = |AD|2 − |AC|2 = .
3

1.3. From the first equality we get zx + xy = 3(x + y + z). If we write down the other
two equations in the same way, we get the system in a slightly different form:

zx + xy = 3(x + y + z), (1)


xy + yz = 5(x + y + z), (2)
yz + zx = 7(x + y + z). (3)

We can solve the system by xy, yz and zx in the following way. If we add (1) and
(2), and subtract (3) from that sum, we get 2xy = x + y + z. In a similar way we get
2yz = 9(x + y + z) and 2zx = 5(x + y + z).
Hence, we have x + y + z = 2xy = 92 yz = 25 zx.
Since x 6= 0, y 6= 0 and z 6= 0 hold from the given system of equations, we conclude
59
9x = 5y = z. By plugging this into the first equation we get x = . Analogously,
18
59 59
we get y = ,z= . Finally, it is easy to check that
10 2
59 59 59
Å ã
(x, y, z) = , ,
18 10 2
is a solution of the given system of equations.

b 1 b2 − a
1.4. Number c is a positive integer if and only if − = is also a positive
a b ab
2
integer. We conclude that b must divide b − a, so b divides a. Therefore we have
b2 − a b−k
a = kb for some k ∈ N. From this we get = . Hence, b − k must be
ab bk
divisible by b and by k.
We conclude that b divides k and k divides b, so it must be b = k. Hence a = b2 and
we have
b 1 b 1
c = a + − = b2 + 2 − = b2 .
a b b b
Final round – National competition 49

1.5. Let r, b and g be the number of red, blue and green points, respectively. By the
given conditions we have r + b + g = 15, r > b and r > g.
Let R, B and G be some red, blue and green point, respectively. By the triangle
inequality we have
|RB| + |BG| > |RG|.
If we add up these inequalities for all red, blue and green points, we get

5g + 25r > 31b.

Analogously we get
31b + 5g > 25r, 25r + 31b > 5g.

If we plug r = 15 − b − g into the first inequality, we get 375 > 20g + 56b.
Hence, we get 375 > 56b, i.e. b 6 6.
If we plug g = 15 − r − b into the second inequality, we get 75 + 26b > 30r. Since
b 6 6, we get 75 + 156 > 30r, i.e. r 6 7.
If we plug g = 15 − r − b into the first inequality, we get 75 + 20r > 36b. Since r 6 7,
we get 75 + 140 > 36b, i.e. b 6 5. From 75 + 26b > 30r and b 6 5 we get r 6 6.
Furthermore, g = 15 − b − r > 15 − 5 − 6 = 4.
Since 6 > r > b, 6 > r > g > 4 and r + b + g = 15, we get either r = 6, b = 5, g = 4
or r = 6, b = 4, g = 5.
All three inequalities that we obtained for r, b and g are satisfied only by the triple
r = 6, b = 5 and g = 4, so that is the only solution.

∗∗∗

2.1. Let us look at the quadratic function

f (x) = ax2 + 2bx + c.

The conditions of the problem give

a + 2b + c > 0 =⇒ f (1) > 0, and


a − 2b + c < 0 =⇒ f (−1) < 0.

Since f (−1) < 0 and f (1) > 0, function f must have a root between −1 and 1. Since
f is a quadratic equation, it must have two distinct real roots. That means that it
has a positive discriminant:

4b2 − 4ac > 0 =⇒ b2 > ac,

which completes the proof.


50 Mathematical competitions in Croatia in 2016

2.2. We analyse the parity of the numbers a, b and c.


Let 2k be the greatest power of 2 which divides all three numbers a, b and c, where
k > 0. We can write a = 2k x, b = 2k y and c = 2k z for some integers x, y and z.
At least one of the numbers x, y and z is odd. From x+y +z = 0 we can conclude that
exactly one number out of x, y and z is even, and the other two are odd. Therefore,
the expression x2 + y 2 + z 2 gives remainder of 2 modulo 4, i.e. it is divisible by 2, but
it is not divisible by 4.
Since the second condition of the problem gives 22k (x2 + y 2 + z 2 ) = 2m 3n , we get
m = 2k + 1 for some non-negative integer k.
Finally, we construct the example which shows that all pairs (m, n) where m is odd
and n is an arbitrary positive integer are indeed the solutions:

• m = 2k + 1, n = 2l is attained by a = 2k 3l , b = −2k 3l , c = 0
• m = 2k + 1, n = 2l − 1 is attained by a = b = 2k 3l−1 , c = −2k+1 3l−1 .

2.3. Let us denote ^BAC = α, ^CBA = β and ^ACB = γ.


Let I 0 be the intersection of the bisector of the angle ^BAC and the line DE. It is
enough to prove that I 0 lies on the bisector of the angle ^BCA, i.e. that ^ACI 0 = 21 γ.

t
I′
D
B C

Since t is tangent to the circumcircle of ABC, we have ^DAF = ^BAF = γ.


The angle ^DAF is central, and the angle ^DEF is inscribed and subtended by the
chord DF , so we get ^DEA = ^DEF = 21 γ. Analogously, ^DEC = 12 ^DAC = 12 α.
Since AI 0 is the bisector of the angle ^BAC, we have ^I 0 AC = 21 α. Since ^I 0 AC =
^I 0 EC, we conclude that the quadrilateral AI 0 CE is cyclic. Therefore,
1
^ACI 0 = ^AEI 0 = ^F ED = γ.
2
Final round – National competition 51

2.4. By adding up all three equations we get


1 1 1
x3 + 2x2 + + y 3 + 2y 2 + + z 3 + 2z 2 + = 3. (*)
4x 4y 4z
1
We will prove x3 + 2x2 + > 1 for all x > 0. Multiplying by 4x (a positive number),
4x
we see that the inequality is equivalent to 4x4 +8x3 +1 > 4x, i.e. to (2x2 +2x−1)2 > 0,
which obviously holds.
In order for the left hand side in (*) to be equal to 3, in the proved inequality the
equality has to occur, i.e. 2x2 + 2x − 1 = 0 must hold.
√ √
−1 ± 3 −1 + 3
Solving that equation we get x = . Since x is positive, we get x = .
2 2 √
−1 + 3
The analogous conclusion holds for the remaining unknowns, so x = y = z =
2
is the only solution.

2.5. Let us write all numbers from 1 to 2017 in the increasing order in each row of
the given board 2016 × 2017. By removing the last cells in some two rows we remove
numbers 2017 and 2017.
Each tile covers five numbers whose sum is divisible by 5: the cross covers numbers
n, n − 1, n, n + 1 and n whose sum is equal to 5n, while rectangle 5 × 1 covers either
5 same or 5 consecutive numbers. If the required tiling was possible, the sum of all
numbers covered with tiles would be divisible by 5.
For the initial sum on the board we have
2017 · 2018
· 2016 = 2017 · 2018 · 1008 ≡ 2 · 3 · 3 ≡ 3 (mod 5).
2
By removing the last cells in some two rows we get the sum that leaves remainder of
4 when divided by 5. Hence, the required tiling is not possible.

∗∗∗

3.1. Let E be the point such that the triangle BDE is right and isosceles with the
right angle at D, and E and A are on the same side of the line BD
√ (i.e. let E be the
point obtained by rotating B around D by 90◦ ). Then |BE| = d 2.

D C

d
b

d
β
A a B


d 2

E
52 Mathematical competitions in Croatia in 2016

Then ^EDA = 90◦ − ^ADB = ^BDC. Since |CD| = |AD| and |DE| = |DB|, by
the S–A–S rule, triangles EAD and BCD are congruent. Therefore |AE| = b.
It follows that

^EAB = 360◦ − ^EAD − ^DAB


= 360◦ − ^DCB − ^DAB = ^ADC + ^ABC = 90◦ + β.

By applying the cosine theorem to the triangle ABE we have

2d2 = a2 + b2 − 2ab cos(90◦ + β) = a2 + b2 + 2ab sin β.

3.2. Assume the opposite, i.e. there exist positive integers m and n such that

k + 4 = m3 and k 2 + 5k + 2 = n3 .

Then the product

(mn)3 = (k + 4)(k 2 + 5k + 2) = k 3 + 9k 2 + 22k + 8

is also a cube of a positive integer.


Since

(k + 2)3 = k 3 + 6k 2 + 12k + 8 < k 3 + 9k 2 + 22k + 8 < k 3 + 9k 2 + 27k + 27 = (k + 3)3 ,

we have that (mn)3 is between two consecutive complete cubes, which is impossible.
Therefore, such k does not exist.

3.3. The left side of the inequality can be written as


1 1 1
Å ã
3 3 3
x +y +z +2 + 3+ 3 .
x3 y z

By the AM–GM inequality, it follows


 
1 1 1 1
+ 3 + 3 > 3 3 3 3 3 = 3.
x3 y z x y z

Therefore
x6 + 2 y 6 + 2 z 6 + 2 1 1 1
+ + > x3 + y 3 + z 3 + 3 + 3 + 3 + 3
x3 y3 z3 x y z
1 1 1
Å ã Å ã Å ã
= x3 + 3 + 1 + y 3 + 3 + 1 + z 3 + 3 + 1
y z x
  … …
x 3 3 3
3 y 3 z
AM–GM
> 33 3 +3 +3
y z3 x3
x y z
Å ã
=3 + + .
y z x
Final round – National competition 53

3.4. Let E be the intersection of lines BH and AC.

S
Q

M P
E
H

B D C

Then ^SBD = 90◦ − 12 ^DSB = 90◦ − ^P HE = ^EP H = 180◦ − ^AP D.


If M is the midpoint of segment AD, then M , Q and S are collinear and we have
^DQM = 12 ^AQD = 180◦ − ^AP D = ^SBD.
Therefore, BDQS is cyclic.

3.5. No. We can write numbers 1 through 2017 (in increasing order) on the fields
in each row. By removing two fields from the last column, we are removing numbers
2017 and 2017.
Any tile covers five numbers whose sum is divisible by 5: the cross covers numbers n,
n − 1, n, n + 1 and n whose sum is 5n, while the 5 × 1 rectangle covers either 5 same
or 5 consecutive numbers.
2017 · 2018
The initial sum · 2016 = 2017 · 2018 · 1008 gives a remainder 2 · 3 · 3 ≡ 3
2
when divided by 5. By removing two fields from the last column, the remaining total
sum gives remainder 4 when divided by 5. The required tiling is not possible, because
the sum of all fields covered by the tiles should be divisible by 5.

∗∗∗

4.1. Denote f (0) = a and f (1) = b.


By plugging x = 0 into the given equation, we have

b = (a − 1)f (y) + 2.

b−2
For a 6= 1 it follows that f (y) = for all y ∈ R. By plugging this into the starting
a−1
equation, we can see that the constant function is not a solution.
Therefore a = 1, so b = 2.
54 Mathematical competitions in Croatia in 2016

By plugging y = 0 into the starting equation, we have


b = f (x) · a − a − x + 2, i.e. 2 = f (x) − x + 1.
It follows that f (x) = x + 1.
We can verify that this is indeed the solution.

4.2. Using mathematical induction, we will show that the numbers in the nth row
make an arithmetic sequence with the difference 2n−1 .
For n = 2 the claim holds, which can be seen directly from the fact that the second
row contains consecutive odd integers.
Assume the claim holds for n − 1. Let A, B and C be three consecutive numbers from
the (n − 1)th row, and A + B, B + C consecutive numbers in the nth row. By the
assumption of the induction, we have B − A = C − B = 2n−2 , so (B + C) − (A + B) =
(C − B) + (B − A) = 2n−2 + 2n−2 = 2n−1 . This shows that the numbers in the nth
row make an arithmetic sequence with the difference 2n−1 .
Let an be the first number in the nth row. For all n ∈ {1, 2, . . . , 2016} we have
an = an−1 + an−1 + 2n−2 = 2an−1 + 2n−2 .

Using this relation and a1 = 1, we can easily show by induction that an = (n+1)2n−2 .
Therefore, the required number is a2016 = 22014 · 2017.

4.3. Let G be the intersection of the circle circumscribed to the triangle DAC and
the diagonal BD. Then
^BGC = 180◦ − ^CGD = 180◦ − ^CAD = 180◦ − 66◦ = 114◦ = ^BAD.

A B

Since ^CBG = ^GBA, it follows that the triangles CGB and DAB have the same
angles, therefore
^GCB = ^ADB.
Final round – National competition 55

The quadrilateral AGCD is cyclic, so we can conclude that ^ACG = ^ADG =


^GCB. Therefore, G lies on the bisector of angle ^ACB, so G is the centre of the
circle inscribed in ABC.
Since AGCD is cyclic and AG is the bisector of angle ^BAC, it follows that
1
^BDC = ^GDC = ^GAC = ^BAC = 24◦ .
2

4.4. Let m, n and k be positive integers that satisfy the given equation.
Since 7n = k 2 − 3m is divisible by 7, k 2 and 3m must have the same remainder when
divided by 7, which is possible only if m is even.
Therefore, m = 2l for some positive integer l, so we can write

7n = (k − 3l )(k + 3l ).

From the above equation we have that both factors are powers of 7, i.e.

k − 3l = 7a ,
k + 3l = 7b ,

where a and b are some non-negative integers.


By subtracting the first equation from the second, we have

2 · 3l = 7a (7b−a − 1).

Since 2 · 3l is not divisible by 7, it follows that a = 0 and

1 + 2 · 3l = 7b .

For l = 1 we have that m = 2 and n = 1, so k = 4.


If l > 2, then 7b = 1 + 2 · 3l gives remainder 1 when divided by 9.
By considering the remainders that powers of 7 give when divided by 9, we conclude
that b must be divisible by 3, i.e. b = 3s for some positive integer s.
From the formula for difference of powers, 7b − 1 must be divisible by 73 − 1 = 342 =
2 · 32 · 19. Therefore, 19 divides 7b − 1. On the other hand, it is not possible for
2 · 3l = 7b − 1 to be divisible by 19.
The only solution is (2, 1, 4).

4.5. Let An (resp. Bn ) be the number of all possible orders of n cyclists at the end
of a race in which each cyclist overtook exactly (resp. at most) once during the race.
We can mark the cyclists in the initial order with numbers 1 through n.
Let us first observe the order after a race in which each cyclist overtook at most once.
The cyclist marked by n (who started the game in last position) can finish last or
next to last, because he can not overtake more than once:
56 Mathematical competitions in Croatia in 2016

• if n finishes last, than that order can be obtained by only having the first n − 1
cyclists overtake – there are Bn−1 such orders.
• if n finished next to last, than that order can be obtained by having n be the last
one to overtake, and before him only the first n − 1 cyclists (or some of them)
have overtaken – there are Bn−1 such orders.
This shows that Bn = Bn−1 + Bn−1 = 2Bn−1 . Since B2 = 2, we have that B =
B200 = 2199 .
Let us now observe the order after a race in which each cyclist overtook exactly once.
Let k be the smallest number such that cyclist k + 2 overtook before cyclist k + 1.
Then the final order must be 2, 3, . . . , k − 1, 1, k, x, . . . , y because cyclist k + 2
made it impossible for cyclists marked 1 through k to mix with cyclists with marks
bigger than k, so the only possibility is for cyclists 1 through k to have overtaken in
the following order: 2, 3, . . . , k, 1. This shows that the final order is determined by
the selection of the number k (which can be any of the numbers 0 through n − 2) and
the order in which we select marks k + 1 through n (the number of these selections
is the same as the number of different final orders in a race with n − k cyclists, i.e.
An−k ), i.e. we have that

An = An−1 + An−2 + · · · + A2 + 1.

Since A2 = 1, it can be easily shown by induction that An = 2n−2 for all positive
integers n. Therefore, 2A = 2A200 = 2199 = B.
II.4 Croatian Mathematical Olympiad

Day 1

A1. Suppose that a sequence a1 , a2 , . . . of positive real numbers satisfies


kak
ak+1 >
a2k + (k − 1)
for every positive integer k. Prove that a1 + a2 + · · · + an > n for every n > 2.
(IMO Shortlist 2015)

C1. Let A be a set containing 2016 points in the plane no four of which are collinear.
Show that it is possible to choose a subset B ⊂ A containing at least 63 points
such that no three points of B are collinear. (Paul Erdős)

G1. Let ABCD be a cyclic quadrilateral such that the tangents to its circumscribed
circle k at points B and D intersect on line AC. Let E and F be points on
circle k such that lines AC, DE and BF are parallel. Let M be the intersection
of BE and DF . If P , Q and R are the feet of the altitudes of triangle ABC,
prove that points P , Q, R and M all lie on the same circle. (USAJMO 2011)

N1. Let m and n be positive integers such that m > n. Define


m+k
xk = ,
n+k
for k = 1, 2, . . . , n+1. Prove that if all the numbers x1 , x2 , . . . , xn+1 are integers,
then
x1 x2 · · · xn+1 − 1
is divisible by an odd prime. (IMO Shortlist 2015)

∗∗∗

Day 2

A2. Let x1 , x2 , . . . , xn > 0 be non-negative real numbers. Prove that the following
inequality holds:
 x2 xn  (n + 1)2 2
x1 + + ··· + (x1 + 2x2 + · · · + nxn ) 6 (x1 + x2 + · · · + xn ) .
2 n 4n
(Estonia 2013)

57
58 Mathematical competitions in Croatia in 2016

C2. An N × N (N > 2) board is given such that two diagonally opposite corner
squares are coloured black, while the remaining squares are left white. In each
step, we choose a row or column and invert the colour of every square it contains.
What is the least number of squares that initially need to be coloured black in
addition to the two corner squares, so that after a finite number of steps we can
obtain a board which is entirely black. (Moscow Olympiad 2007)

G2. Let P be a point inside a triangle ABC such that


|AP | + |BP | |BP | + |CP | |CP | + |AP |
= = .
|AB| |BC| |CA|
Let A0 , B 0 and C 0 be the points (different from A, B, C) at which the circum-
scribed circle of ABC intersects lines AP , BP and CP , respectively. Prove that
the incircles of triangles ABC and A0 B 0 C 0 coincide. (Vjekoslav Kovač)

N2. Find all pairs (p, q) of prime numbers such that


p(p2 − p − 1) = q(2q + 3). (Belarus 2015)

∗∗∗

Final test for MEMO team selection

A3. Suppose f : R → R satisfies


f (xf (y)) + f (x2 ) = f (x)(x + f (y)), for all x, y ∈ R,
2016
and f (2) = 3. Find f (2 ). (Tonći Kokan)

C3. Can a 12 × 12 board be covered by L-triominoes so that each row and each
column intersects the same number of triominoes?
L-triomino consists of three 1 × 1 squares, not all three in the same row or the
same column. (Russia 2013)

G3. Let ABCD be a cyclic quadrilateral. Rays AD and BC meet at P . In the


interior of the triangle DCP a point M is given, such that the line P M bisects
the angle ^CM D. Line CM meets circumcircle of DM P again in Q, and the
line DM meets circumcircle of CM P again in R.
a) Prove that segments CQ and DR have equal lengths.
b) Prove that triangles P AQ and P BR have equal areas.
(Geometry from the Land of the Incas)

N3. Find all pairs (m, n) of positive integers such that


3 · 5m − 2 · 6n = 3. (Sweden 2014)

∗∗∗
Croatian Mathematical Olympiad 59

Final test for IMO team selection

A4. Find all functions f : R → R that satisfy the condition

f (x2 ) + xf (y) = f (x)f (x + f (y)), for all x, y ∈ R.

(Tonći Kokan)

C4. There are N > 3 points given in a plane such that no three points are collinear.
Each line segment connecting two of these points is coloured red or blue. Prove
that there exist N − 1 line segments of the same colour such that they don’t
divide the plane into more than one area and that they don’t intersect each
other (except possibly in endpoints). (Theorem in Ramsey Theory)

G4. Let point O be the circumcentre of the acute triangle ABC. Let points E and
F lie on the segments OB and OC, respectively, so that |BE| = |OF |. Let
M and N be midpoints of the arcs EOA˙ and AOF ˘, respectively. Prove that
^EN O + ^OM F = 2^BAC. (Ukraine 2013)

N4. Let p > 109 be a prime number such that 4p+1 is also prime. Prove that decimal
1
representation of the number contains all the digits 0, 1, . . . , 9.
4p + 1
(Bulgaria TST 2014)

II.4.1 Solutions

Day 1

A1. From
kak
ak+1 > , (1)
a2k + (k − 1)
it can be seen that
k a2k + (k − 1) k−1
6 = ak + ,
ak+1 ak ak
and so
k k−1
ak > − .
ak+1 ak
Summing the above inequality for k = 1, . . . , m, we obtain
1 0 2 1  m m − 1 m
a1 + a2 + · · · + am > − + − +···+ − = . (2)
a2 a1 a3 a2 am+1 am am+1
Now we prove the problem statement by induction on n. The case n = 2 can be done
by applying (1) to k = 1:
1
a1 + a2 > a1 + > 2.
a1
60 Mathematical competitions in Croatia in 2016

For the induction step, assume that the statement is true for some n > 2. If an+1 > 1,
then the induction hypothesis yields
(a1 + · · · + an ) + an+1 > n + 1.
Otherwise, if an+1 < 1 then apply (2) as
n n−1  1 
(a1 + · · · + an ) + an+1 > + an+1 = + + an+1 > (n − 1) + 2 = n + 1.
an+1 an+1 an+1
That completes the solution.

C1. Let B ⊆ A be the largest subset which does not contain three collinear points.
Let |B| = k.
Since B is the largest set with this property, any point in A\B lies on some line which
passes through two points of B. On the other hand, every line which passes through
two points of B can contain at most one point of A \ B because of the assumption
that no four points of A lie on the same line. This means that the number of points
in A \ B is less than or equal to the number of pairs of points in B.
The number of points in A \ B is 2016 − k, and the number of pairs of points in B
equals k(k−1)
2 . Thus, we have
k(k − 1)
2016 − k 6 .
2
This inequality holds if and only if k > 63 or k 6 −64. Since k must be positive, we
conclude that B contains at least 63 points.

G1. It is well-known that the feet of the altitudes and the midpoints of the sides of
triangle ABC all lie on the same circle (called the nine-point circle or the Feuerbach’s
circle). Therefore, it suffices to prove that M is the midpoint of AC.
Let M 0 be the intersection of lines BE and AC. We shall prove that M 0 is the
midpoint of AC.

E D

M = M′
A C X

F B
Croatian Mathematical Olympiad 61

Let X be the intersection of line AC with the tangents at B and D. ACDE is a


cyclic trapezium and we have |CD| = |AE| and ^DCM 0 = ^M 0 AE. If we prove that
^EM 0 A = ^CM 0 D, i.e. ^XM 0 B = ^XM 0 D, then we will have (by A–S–A) that the
triangles AM 0 E and CM 0 D are congruent. This would imply that |AM 0 | = |CM 0 |.
Let us prove that this is indeed the case. Using the tangent–chord theorem and the
fact that AC and DE are parallel, we get
^XM 0 B = ^DEB = ^XDB,
which shows that XDM 0 B is cyclic. This, along with the fact that the tangent
segments XB and XD have equal lengths, implies
^XM 0 D = ^XBD = ^XDB = ^XM 0 B.
Therefore, the line BE passes through the midpoint of AC. An analogous argument
shows that DF passes through the midpoint of AC. Since two different lines can have
at most one point in common, we conclude that M is the midpoint of AC, completing
the proof.

N1. Assume that x1 , x2 , . . . , xn+1 are integers. Define the integers


m+k m−n
ak = xk − 1 = −1= >0
n+k n+k
for k = 1, 2, . . . , n + 1.
Let P = x1 x2 · · · xn+1 − 1. We need to prove that P is divisible by an odd prime, or
in other words, that P is not a power of 2. To this end, we investigate the powers of
2 dividing the numbers ak .
Let 2d be the largest power of 2 dividing m − n, and let 2c be the largest power of 2
not exceeding 2n + 1. Then 2n + 1 6 2c+1 − 1, and so n + 1 6 2c . We conclude that
2c is one of the numbers n + 1, n + 2, . . . , 2n + 1, and that it is the only multiple of
2c appearing among these numbers. Let l be such that n + l = 2c . Since m−n n+l is an
integer, we have d > c. Therefore,
m−n
2d−c+1 - al = ,
n+l
while
2d−c+1 | ak for all k ∈ {1, . . . , n + 1} \ {l}.
d−c+1
Computing modulo 2 , we get
P = (a1 + 1)(a2 + 1) · · · (an+1 + 1) − 1 ≡ (al + 1) · 1n − 1 ≡ al 6≡ 0 (mod 2d−c+1 ).
Therefore, 2d−c+1 - P .
On the other hand, for any k ∈ {1, . . . , n + 1} \ {l}, we have 2d−c+1 | ak . So
P > ak > 2d−c+1 ,
and it follows that P is not a power of 2.

∗∗∗
62 Mathematical competitions in Croatia in 2016

Day 2

A2. We have
n
! n
! n
! n
!
X xk X 1 X nxk X
· kxk = · · kxk
k n k
k=1 k=1 k=1 k=1
n n
!2 n
!2
GM–AM 1 1 X nxk X 1 X n 
6 · + kx = xk +k
n 4 k 4n k
k=1 k=1 k=1
n
!2
(n + 1)2 X
6 xk .
4n
k=1

The last inequality follows from


n
+ k 6 n + 1 ⇐⇒ (n − k)(k − 1) > 0.
k
Note that equality holds if and only if x1 = xn and x2 = · · · = xn−1 = 0.

C2. Notice that the order of row and column operations is not relevant, so we may
assume that all the row operations are performed before manipulating any of the
columns. Furthermore, we observe that it is sufficient to transform any row or column
at most once.
Let as analyze the initial colouring of the board which can, after a finite number of
moves, become entirely black. If we are to perform column operations only after all
the row operations have been done, we see that before we start manipulating the
columns, all the rows must have the same appearance.
We conclude that, initially, all the rows must be coloured in one of the two possible
ways which can be obtained from one another by applying the allowed row transform-
ation.
If every row in the initial configuration contains at least two black squares, then the
total number of black squares is at least 2n. Assume that there is a row which contains
at most one black square. This row cannot be equal to both the first row and the
last, because they already have black squares placed at two different positions – the
first and the last, respectively. This means that there is a row which has the opposite
colouring, thus containing at least n − 1 black squares. If this row contains n black
squares, then both the first and the last row must be entirely black, yielding a total
number of black squares larger than 2n. If this row contains n − 1 black squares, then
the remaining rows must contain at least 1 black square, which means that there are
at least 2n − 2 black squares in total.
This shows that the least number of squares that initially need to be coloured black
is 2n − 2, meaning we have to add at least 2n − 4 black squares.
On the other hand, if we colour all the squares in the first column and in the last
row black, leaving only the bottom–left corner white (as shown in the picture), we
can perform the obvious operations (transforming the first n − 1 rows, then the first
Croatian Mathematical Olympiad 63

n − 1 columns) and colour the entire board black. Since this colouring requires 2n − 4
additional black squares, we have shown that the least number of squares we need to
colour black is equal to 2n − 4.

G2. We denote the side lengths of triangle ABC by a, b and c, its semiperimeter by
s, the radii of the incircle and the circumcircle by r and R respectively, and its area
by p. The corresponding elements of triangle A0 B 0 C 0 are denoted by the same letters
with an additional prime symbol (0 ). We start by showing that the triangles ABC
and A0 B 0 C have equal radii of the incircles.

B′

B′

C′ I = I′ D′
P C′
P I′
B C I

A′ B D C

If we denote by λ the common value of the fractions appearing in the definition of P ,


we have

|AP | + |BP | = λc, |BP | + |CP | = λa, |CP | + |AP | = λb.

Solving this in terms of |AP |, |BP | and |CP |, we obtain

|AP | = λ(s − a), |BP | = λ(s − b), |CP | = λ(s − c). (3)

Multiplying these three equalities, using Heron’s formula and the fact that p = rs we
arrive at
|AP ||BP ||CP | = λ3 (s − a)(s − b)(s − c) = λ3 p2 /s = λ3 pr. (4)
Furthermore, triangles P BC and P C 0 B 0 are similar because, by the inscribed angle
theorem, they have three pairs of congruent angles (subtending BC 0 and B 0 C). From
here we have
a0 |B 0 C 0 | |C 0 P | |B 0 P |
= = = . (5)
a |BC| |BP | |CP |
Multiplying this with the two relations obtained analogously, we get

a0 b0 c0 |A0 P ||B 0 P ||C 0 P |


= . (6)
abc |AP ||BP ||CP |
64 Mathematical competitions in Croatia in 2016

On the other hand, using formula p = abc/4R and the fact that triangles ABC and
A0 B 0 C 0 have the same circumcircle, we obtain
a0 b0 c0 p0
= . (7)
abc p

From (5), we also have


|B 0 P | + |C 0 P | |BP | + |CP |
= =λ
|B 0 C 0 | |BC|
and, analogously,
|A0 P | + |B 0 P | |C 0 P | + |A0 P |
= = λ.
|A0 B 0 | |C 0 A0 |
It follows that P has the same property with respect to triangle A0 B 0 C 0 as with the
original triangle ABC (moreover, with the same ratio λ). Thus, an equality analogous
to (4) must hold for A0 B 0 C 0 :
|A0 P ||B 0 P ||C 0 P | = λ3 p0 r0 . (8)
Combining (4), (6), (7) and (8) yields
p0 p0 r 0
= ,
p pr
which finally allows us to conclude r0 = r. Let us denote by I the centre of the incircle
of triangle ABC, and by D the point at which it touches BC. Points I 0 and D0 are
defined analogously with respect to triangle A0 B 0 C 0 . It is well-known that
|BD| = s − b, |CD| = s − c,
which, combined with (3), implies
|BP | |BD|
= .
|CP | |CD|
By the angle bisector theorem, we conclude that P D is the bisector of ^BP C. Using
the corresponding relations with respect to A0 B 0 C 0 we also conclude that P D0 is the
bisector of ^B 0 P C 0 . Thus, points P , D and D0 are collinear. Furthermore, we have
1
^C 0 D0 P = 180◦ − ^D0 P C 0 − ^P C 0 D0 = 180◦ − ^B 0 P C 0 − ^CC 0 B 0
2
◦ 1 0 ◦
= 180 − ^BP C − ^CBB = 180 − ^BP D − ^DBP = ^P DB.
2
Since I 0 D0 ⊥ B 0 C 0 and ID ⊥ BC, we have obtained ^DD0 I 0 = ^IDD0 . Let us recall
that |D0 I 0 | = r0 = r = |DI|. If we had I 6= I 0 , either D0 DII 0 or D0 DI 0 I would be
an isosceles trapezium. This would imply that II 0 is parallel to P D. In the same
manner, we would conclude that II 0 is parallel to the remaining two lines passing
through P and one of the points at which the incircle touches the sides of the triangle
ABC. This leads to a contradiction with the fact that, of these three lines, at least
two are different. We have thus proved that I = I 0 , which implies that ABC and
A0 B 0 C share the same incircle.
Croatian Mathematical Olympiad 65

N2. If p = q, we have p2 − p − 1 = 2p + 3, i.e. p2 − 3p − 4. The solutions to this


equation are p1 = −1 and p2 = 4, which aren’t prime numbers.
Let us, from now on, assume that p 6= q. In that case, p divides 2q + 3. Therefore,
there exists a natural number k such that 2q + 3 = kp, i.e. q = kp−3
2 . Plugging this
into the original equation we get

2p2 − 2p − 2 = (kp − 3)k, i.e. 2p2 − (k 2 + 2)p + (3k − 2) = 0.

Treating this as a quadratic equation in terms of p, we conclude that the discriminant


D = (k 2 + 2)2 − 8(3k − 2) must be a perfect square.
Obviously, D < (k 2 + 2)2 . If k > 6, then 2k 2 − 12k + 10 > 0, which implies D > k 4 .
In this case, the only possibility is D = (k 2 + 1)2 , i.e.

(k 2 + 2)2 − 8(3k − 2) = (k 2 + 1)2 ,

which simplifies into 2k 2 − 12k + 11 = 0. This equation has no integer solutions, so


we conclude that k < 6.
Plugging in k = 1, 2, 3, 4, 5, 6, we observe that only k = 5 makes D a perfect square.
Thus, the only solution is k = 5, p = 13, q = 31.

∗∗∗

Final test for MEMO team selection

A3. Putting x = 0 and y = 0 into the given equation gives us 2f (0) = f (0)2 . Hence,
f (0) = 2 or f (0) = 0.
Assume f (0) = 2. Putting x = 1, y = 0 gives us f (f (0)) = f (1)f (0) and 3 = f (2) =
2f (1). Then f (1) = 23 . Putting x = 0, y = 1 gives us 2f (0) = f (0)f (1), so f (0) = 0,
which is a contradiction.
Therefore, f (0) = 0, and putting y = 0 gives us f (x2 ) = f (x)x, so the starting
equation becomes f (xf (y)) = f (x)f (y). Putting y = x, gives us f (xf (x)) = f (x)2 .
Applying f to f (x2 ) = f (x)x and using f (xf (x)) = f (x)2 we get

f (f (x2 )) = f (f (x)x) = f (x)2 .

Supstituting x = 1 in f (xf (y)) = f (x)f (y) we get f (f (y)) = f (1)f (y), which together
with f (f (x2 )) = f (x)2 yields

xf (x)f (1) = f (x2 )f (1) = f (f (x2 )) = f (x)2 .

Therefore, each x satisfies f (x) = xf (1) or f (x) = 0.


Since f (2) 6= 0, we get f (1) = 23 .
1
Putting y = 2, gives us f (3x) = 3f (x), and inductively we get f (32016 ) = 2 · 32017 .
66 Mathematical competitions in Croatia in 2016

Putting y = 1 yields
3 3
Å ã
f x = f (x),
2 2
so f (22016 ) = 3 · 22015 .
We also need to make sure such a function really exists. One such (non-unique)
example is the function f (x) = 23 x.

C3. Assume such a covering is possible. Each triomino intersecting first row must
also intersect second row. Since each row intersects the same number of triominoes,
all the triominoes intersecting second row must intersect the first row, i.e. first two
rows are covered by triominoes (without crossing the line between the second and the
third row). Analogously, we conclude the same for first two columns.
Therefore, 2 × 2 square which is the intersection of the first two rows and the first
two columns is covered without overlapping by triominoes. That is clearly impossible
and our assumption was wrong, so 12 × 12 board cannot be covered in the desired
manner.

G3. Let N be a point on the P M such that M is between P and N .

Q R

C
D

A B

Since M P QD is cyclic, we have

^P DQ = ^P M Q = ^CM N = ^DM N = 180◦ − ^DM P = ^DQP.

Hence, triangle DP Q is isosceles. Analogously we conclude that CP R is isosceles and


similar to DP Q, because of the equality of their angles.
Croatian Mathematical Olympiad 67

a) Equalities |P Q| = |P D|, |P R| = |P C|, and ^CP R = ^DP Q hold. It follows


that ^CP Q = ^DP R. By the S–A–S theorem, triangles RP D and CP Q are
congruent. Therefore, |CQ| = |DR|.
b) Let ^AP Q = ^BP R = ϕ. Following holds

2P (AP Q) = |AP | · |P Q| · sin ϕ and 2P (BP R) = |BP | · |P R| · sin ϕ.

So the areas are equal if |AP | · |P Q| = |BP | · |P R|.


Applying power of a point theorem to P and the circumcircle of ABCD we get
|AP | · |P D| = |BP | · |P C|, then from |P Q| = |P D| and |P R| = |P C| the desired
statement follows.

N3. If n = 1, then 3 · 5m − 12 = 3, so one solution is (m, n) = (1, 1).


For n = n0 + 1 > 2, n0 ∈ N, equation becomes 5m − 2 · 2n0 +1 · 3n0 = 1, i.e.

5m − 1 = 4 · 6n0 .

Since the right hand side is divisible by 6, it follows that m is even, i.e. m = 2m0 ,
m0 ∈ N. Now we have
(5m0 − 1)(5m0 + 1) = 4 · 6n0 .
Considering M (5m0 + 1, 5m0 − 1) = M (5m0 − 1, 2) = 2, we can write

5m0 + 1 = 2 · 3n0 , 5m0 − 1 = 2n0 +1 .

Subtraction and division by 2 gives us 1 = 3n0 −2n0 , which is possible only for n0 = 1.
For n0 > 1, difference on the right hand side is greater than 1. For n0 = 1 we get
m0 = 1, i.e. the only other possible solution is (m, n) = (2, 2).

∗∗∗

Final test for IMO team selection

A4. Putting x = 0 into the given equation gives us f (0) = f (0)f (f (y)), and hence

f (0)[f (f (y)) − 1] = 0, ∀y ∈ R.

In case that f (f (y)) = 1, ∀y ∈ R, we have f (f (0)) = 1 and f (1) = f (f (f (0))) = 1.


Putting x = y = 1 into the given equation gives us f (1) + f (1) = f (1)f (1 + f (1)),
which implies f (2) = 2. However, 1 = f (f (2)) = f (2) = 2, which is a contradiction.
Therefore, f (0) = 0 and putting y = 0 into the given equation yields

f (x2 ) = f (x)2 , ∀x ∈ R. (9)

From the previous equation, we get f (12 ) = f (1)2 and thus f (1) = 0 or f (1) = 1.
68 Mathematical competitions in Croatia in 2016

1. If f (1) = 0, putting x = 1 into the given equation gives us 0 + f (y) = 0, i.e.


f (x) = 0, ∀x ∈ R, which is a solution.
2. If f (1) = 1, putting x = 1 into the given equation gives us 1+f (y) = f (1+f (y)),
i.e.
f (f (x) + 1) = f (x) + 1, ∀x ∈ R. (10)
Putting y = 1 into the given equation, together with (9), gives us

f (x)2 + x = f (x)f (x + 1), ∀x ∈ R, (11)

which implies that f (x) = 0 =⇒ x = 0, i.e. f (x) 6= 0, ∀x ∈ R \ {0}.


Furthermore, by putting y = x into the given equation and using (9) we get
f (x)2 +xf (x) = f (x)f (x+f (x)), ∀x ∈ R. If x 6= 0 we get f (f (x)+x) = f (x)+x.
However, this also holds for x = 0. Therefore,

f (f (x) + x) = f (x) + x, ∀x ∈ R. (12)

Putting y = x − 1 into the given equation, together with (9), gives us:

f (x)2 + xf (x − 1) = f (x)f (x + f (x − 1))


= f (x)f (f (x − 1) + (x − 1) + 1)
(12)
= f (x)f (f (f (x − 1) + (x − 1)) + 1)
(10)
= f (x)[f (f (x − 1) + (x − 1)) + 1]
= f (x)f (f (x − 1) + (x − 1)) + f (x)
(12)
= f (x)[f (x − 1) + (x − 1)] + f (x)
= f (x)f (x − 1) + xf (x),

which implies that [f (x) − x][f (x) − f (x − 1)] = 0, ∀x ∈ R.


If there exists x0 ∈ R such that f (x0 ) = f (x0 − 1) then putting x = x0 − 1 into
(11) gives us f (x0 − 1)2 + x0 − 1 = f (x0 − 1)f (x0 ) and thus x0 = 1. However,
we know that f (x0 ) = f (1) = 1 6= 0 = f (0) = f (x0 − 1) and therefore the
assumption was wrong. Thus, f (x) = x, ∀x ∈ R, which is another solution to
the given functional equation.

Finally, f (x) = 0 and f (x) = x are the only two functions that satisfy given conditions.

C4. Conditions on N − 1 segments (that they do not divide the plane into more than
one area and that they do not intersect each other except in endpoints) can easily be
shown equivalent to a single condition that these segments form a planar spanning
tree (non-selfintersecting connected cycle-free graph). Now the problem is equivalent
to the existence of a single-coloured tree.
We prove the statement by induction on N . If N = 3, statement is obviously true,
because there are three segments connecting three points, and there must be two
segments of the same colour.
Croatian Mathematical Olympiad 69

Let us assume the statement holds for natural numbers less or equal to N . Consider
the convex hull of N +1 points. Assume there exist two segments of different colour, on
the edge of the convex hull, with a common endpoint A. By the inductive assumption,
the other N points have a single-coloured spanning tree, say T . One of the segments
on the edge of the convex hull containing A must be of the same colour as T . Also, it
can’t intersect T . Adding that segment to T gives us a single-coloured tree consisting
of N + 1 vertices.
Without loss of generality, we may assume that all segments on the edge of the
convex hull are red. Introduce coordinates such that all the points have differ-
ent x−coordinates. Assume the points A1 , A2 , . . . , AN +1 are ordered by ascending
x−coordinate.
For K = 2, . . . , N , by the inductive assumption, there exist single-coloured non-
selfintersecting trees LK (for points A1 , . . . , AK ) and RK (for points AK , . . . , AN +1 ).
If for some K trees LK and RK are of the same colour, then LK ∪ RK is a single-
coloured tree for N + 1 points. So let us assume for each K = 2, . . . , N , that LK and
RK are of different colour.
If R2 is red, R2 ∪ {A1 A2 } is a single-coloured tree with N + 1 vertices. So let us
assume R2 and, analogously, LN are blue. Hence, RN and L2 must be red.
Since L2 is red and LN is blue, there must be some K such that LK is red and LK+1
is blue. Then RK+1 is red and there exists a segment XY on the edge of the convex
hull such that X is in LK and Y is in RK+1 , e.g. we can choose points with greatest
y−coordinates in LK and RK+1 for X and Y . Set of segments LK ∪ {XY } ∪ RK+1
is a single-coloured non-selfintersecting tree with N + 1 vertices, which completes the
proof.

G4. Let α = ^BAC, β = ^CBA, and γ = ^ACB.

A′

M O

E
N

A B

Triangles AM E and AOB are isosceles with equal angles since ^AM E = ^AOE =
70 Mathematical competitions in Croatia in 2016

2γ. Therefore, those triangles are similar.


Observe that ^AOM = ^AEM = 90◦ −γ = ^ABE, hence triangles AM O and AEB
are also similar. It follows that |OA| : |OM | = |AB| : |BE|, i.e. |OB| : |OM | = |AB| :
|OF |. Let A0 be the reflection of A across the line BC.
Now we have |OB| : |OM | = |A0 B| : |OF | and
^M OF = ^AOF − ^AOM
= 2β − (90◦ − γ) = β + (90◦ − α)
= ^A0 BC + ^CBO = ^A0 BO.

From S–A–S theorem it follows that triangles A0 BO and F OM are similar, hence
^F M O = ^A0 OB.

Analogously, it is shown that ^EN O = ^A0 OC, so


^EN O + ^F M O = ^A0 OC + ^A0 OB = ^BOC = 2^BAC.

N4. Let q = 4p + 1 and let a ◦ b be the remainder of a modulo b. It suffices to


show that the last digits of the numbers 10k ◦ q, k ∈ N, attain all values from 0 to 9.
Indeed, since q and 10 are coprime, that ü mean that all digits from 0 to 9 occur
ú would
10k
among the last digits of the numbers and those are the same as the digits in
q
1
the decimal representation of .
q
Let S be the set of all non-zero biquadratic residues modulo q, i.e. the set of all
0 < t < q such that the congruence x4 ≡ t (mod q) has a solution. Then S has p
elements. Indeed, since −1 is a quadratic residue modulo q, the 2p non-zero quadratic
residues modulo q can be grouped in pairs (s, −s), and among their squares (which
are precisely the biquadratic residues) there are precisely p distinct ones.
We will show that each element of S is of the form 10k ◦ q for some k. Let d be the
index of q modulo 10. Since d | ϕ(q) = 4p, we have d = p, d = 2p or d = 4p.
Consider the case d = p first. Let 0 6 k < p; then there is a 0 6 j 6 3 such that
k + jp is a multiple of 4 and the number
k+jp
10k ≡ b10 4 c4 (mod q)
is a biquadratic residue. Since the numbers 10k ◦ q for 0 6 k < p are pairwise distinct
and all of them are biquadratic residues, they must coincide with the elements of S.
The cases d = 2p and d = 4p are treated analogously.
Let u be an arbitrary digit.
» Let 0 6 j 6 3pbe such that u + jq ends in 0, 1, 5 or 6.
9
Since q > 10 , we have 4 (j + 1)q − 1 − 4 u + jq > 6. It follows that the interval
[u + jq, (j + 1)q] contains at least six fourth powers. Therefore, it contains at least
one fourth power x4 that ends in the same digit as u + jq. Let s = x4 ◦ q. Then
x4 = s + jq and 10 | x4 − (u + jq) = s − u, i.e. s ends in u, as needed.

You might also like